NURS 104 Exam 4

Lakukan tugas rumah & ujian kamu dengan baik sekarang menggunakan Quizwiz!

A vasoconstrictor used to treat shock is a. adrenaline. b. Nipride. c. Dobutrex. d. adenosine.

ANS: A Adrenaline is a vasoconstrictor, Nipride is a vasodilator, Dobutrex is an inotrope, and adenosine is an antidysrhythmic.

The ventilator variable that causes inspiration is called the a. cycle. b. trigger. c. flow. d. pressure.

ANS: B The phase variable that initiates the change from exhalation to inspiration is called the trigger. Breaths may be pressure triggered or flow triggered based on the sensitivity setting of the ventilator and the patient's inspiratory effort or time triggered based on the rate setting of the ventilator.

In which order will the nurse take these actions when doing a dressing change for a partial-thickness burn wound on a patient's chest? (Put a comma and a space between each answer choice [A, B, C, D, E].) a. Apply sterile gauze dressing. b. Document wound appearance. c. Apply silver sulfadiazine cream. d. Administer IV fentanyl (Sublimaze). e. Clean wound with saline-soaked gauze.

ANS: D, E, C, A, B Because partial-thickness burns are very painful, the nurse's first action should be to administer pain medications. The wound will then be cleaned, antibacterial cream applied, and covered with a new sterile dressing. The last action should be to document the appearance of the wound. DIF: Cognitive Level: Apply (application) REF: 465

The nurse estimates the extent of a burn using the rule of nines for a patient who has been admitted with deep partial thickness burns of the anterior trunk and the entire left arm. What percentage of the patient's total body surface area (TBSA) has been injured?

ANS: 27% When using the rule of nines, the anterior trunk is considered to cover 18% of the patient's body and each arm is 9%. DIF: Cognitive Level: Understand (comprehension) REF: 454

A patient arrives in the emergency department with facial and chest burns caused by a house fire. Which action should the nurse take first? a. Auscultate the patient's lung sounds. b. Determine the extent and depth of the burns. c. Infuse the ordered lactated Ringer's solution. d. Administer the ordered hydromorphone (Dilaudid).

ANS: A A patient with facial and chest burns is at risk for inhalation injury, and assessment of airway and breathing is the priority. The other actions will be completed after airway management is assured. DIF: Cognitive Level: Apply (application) REF: 452 | 455

Norepinephrine (Levophed) has been prescribed for a patient who was admitted with dehydration and hypotension. Which patient data indicate that the nurse should consult with the health care provider before starting the norepinephrine? a. The patient's central venous pressure is 3 mm Hg. b. The patient is in sinus tachycardia at 120 beats/min. c. The patient is receiving low dose dopamine (Intropin). d. The patient has had no urine output since being admitted.

ANS: A Adequate fluid administration is essential before administration of vasopressors to patients with hypovolemic shock. The patient's low central venous pressure indicates a need for more volume replacement. The other patient data are not contraindications to norepinephrine administration. DIF: Cognitive Level: Apply (application) REF: 1644

When opening the airway of an unresponsive trauma patient in the emergency department, which of the following considerations is correct? a. Airway assessment must incorporate cervical spine immobilization. b. Hyperextension of the neck is the only acceptable technique. c. Flexion of the neck protects the patient from further injury. d. Airway patency takes priority over cervical spine immobilization.

ANS: A Airway assessment must incorporate cervical spine immobilization. The patient's head should not be rotated, hyperflexed, or hyperextended to establish and maintain an airway. The cervical spine must be immobilized in all trauma patients until a cervical spinal cord injury has been definitively ruled out.

Which patient is most appropriate for the burn unit charge nurse to assign to a registered nurse (RN) who has floated from the hospital medical unit? a. A 34-year-old patient who has a weight loss of 15% from admission and requires enteral feedings. b. A 67-year-old patient who has blebs under an autograft on the thigh and has an order for bleb aspiration c. A 46-year-old patient who has just come back to the unit after having a cultured epithelial autograft to the chest d. A 65-year-old patient who has twice-daily burn debridements and dressing changes to partial-thickness facial burns

ANS: A An RN from a medical unit would be familiar with malnutrition and with administration and evaluation of response to enteral feedings. The other patients require burn assessment and care that is more appropriate for staff who regularly care for burned patients. DIF: Cognitive Level: Analyze (analysis) REF: 15-16

Which route for ETT placement is used in an emergency? a. Orotracheal b. Nasotracheal c. Nasopharyngeal d. Trachea

ANS: A An endotracheal tube (ETT) may be placed through the orotracheal or the nasotracheal route. In most situations involving emergency placement, the orotracheal route is used because it is simpler and allows the use of a larger diameter ETT. Nasotracheal intubation provides greater patient comfort over time and is preferred in patients with a jaw fracture.

Which information about a patient who is receiving cisatracurium (Nimbex) to prevent asynchronous breathing with the positive pressure ventilator requiresimmediate action by the nurse? a. Only continuous IV opioids have been ordered. b. The patient does not respond to verbal stimulation. c. There is no cough or gag when the patient is suctioned. d. The patient's oxygen saturation fluctuates between 90% to 93%.

ANS: A Because neuromuscular blockade is extremely anxiety provoking, it is essential that patients who are receiving neuromuscular blockade receive concurrent sedation and analgesia. Absence of response to stimuli is expected in patients receiving neuromuscular blockade. The oxygen saturation is adequate. DIF: Cognitive Level: Apply (application) REF: 1664

The urgent care center protocol for tick bites includes the following actions. Which action will the nurse take first when caring for a patient with a tick bite? a. Use tweezers to remove any remaining ticks. b. Check the vital signs, including temperature. c. Give doxycycline (Vibramycin) 100 mg orally. d. Obtain information about recent outdoor activities.

ANS: A Because neurotoxic venom is released as long as the tick is attached to the patient, the initial action should be to remove any ticks using tweezers or forceps. The other actions are also appropriate, but the priority is to minimize venom release. DIF: Cognitive Level: Apply (application) REF: 1697

A patient with septic shock has a BP of 70/46 mm Hg, pulse 136, respirations 32, temperature 104° F, and blood glucose 246 mg/dL. Which intervention ordered by the health care provider should the nurse implement first? a. Give normal saline IV at 500 mL/hr. b. Give acetaminophen (Tylenol) 650 mg rectally. c. Start insulin drip to maintain blood glucose at 110 to 150 mg/dL. d. Start norepinephrine (Levophed) to keep systolic blood pressure >90 mm Hg.

ANS: A Because of the low systemic vascular resistance (SVR) associated with septic shock, fluid resuscitation is the initial therapy. The other actions also are appropriate, and should be initiated quickly as well. DIF: Cognitive Level: Apply (application) REF: 1644-1645

The main cause of cardiogenic shock is a. an inability of the heart to pump blood forward. b. hypovolemia, resulting in decreased stroke volume. c. disruption of the conduction system when reentry phenomenon occurs. d. an inability of the heart to respond to inotropic agents.

ANS: A Cardiogenic shock is the result of failure of the heart to effectively pump blood forward. It can occur with dysfunction of the right or the left ventricle or both. The lack of adequate pumping function leads to decreased tissue perfusion and circulatory failure.

A nurse is caring for a patient who is orally intubated and receiving mechanical ventilation. To decrease the risk for ventilator-associated pneumonia, which action will the nurse include in the plan of care? a. Elevate head of bed to 30 to 45 degrees. b. Suction the endotracheal tube every 2 to 4 hours. c. Limit the use of positive end-expiratory pressure. d. Give enteral feedings at no more than 10 mL/hr.

ANS: A Elevation of the head decreases the risk for aspiration. Positive end-expiratory pressure is frequently needed to improve oxygenation in patients receiving mechanical ventilation. Suctioning should be done only when the patient assessment indicates that it is necessary. Enteral feedings should provide adequate calories for the patient's high energy needs. DIF: Cognitive Level: Apply (application) REF: 1668

Place these pathophysiologic mechanisms of DIC in the order in which they occur. 1. Activation of the fibrinolytic system 2. Breakdown of thrombi; spontaneous hemorrhage 3. Consumption of coagulation factors; failure of regulatory mechanisms 4. Endothelial damage; release of tissue factor 5. Thrombin formation; clots form along epithelial walls a. 4, 1, 5, 2, 3 b. 4, 1, 3, 2, 5 c. 5, 1, 2, 4, 3 d. 2, 3, 1, 5, 4

ANS: A Endothelial damage triggers the release of tissue factor. Thrombin is released, and clots form, consuming coagulation factors. The fibrinolytic system is triggered to break down the clots, which leads to spontaneous hemorrhage.

A patient has just been admitted with a 40% total body surface area (TBSA) burn injury. To maintain adequate nutrition, the nurse should plan to take which action? a. Insert a feeding tube and initiate enteral feedings. b. Infuse total parenteral nutrition via a central catheter. c. Encourage an oral intake of at least 5000 kcal per day. d. Administer multiple vitamins and minerals in the IV solution.

ANS: A Enteral feedings can usually be initiated during the emergent phase at low rates and increased over 24 to 48 hours to the goal rate. During the emergent phase, the patient will be unable to eat enough calories to meet nutritional needs and may have a paralytic ileus that prevents adequate nutrient absorption. Vitamins and minerals may be administered during the emergent phase, but these will not assist in meeting the patient's caloric needs. Parenteral nutrition increases the infection risk, does not help preserve gastrointestinal function, and is not routinely used in burn patients. DIF: Cognitive Level: Apply (application) REF: 463

A 78-kg patient with septic shock has a urine output of 30 mL/hr for the past 3 hours. The pulse rate is 120/minute and the central venous pressure and pulmonary artery wedge pressure are low. Which order by the health care provider will the nurse question? a. Give PRN furosemide (Lasix) 40 mg IV. b. Increase normal saline infusion to 250 mL/hr. c. Administer hydrocortisone (Solu-Cortef) 100 mg IV. d. Titrate norepinephrine (Levophed) to keep systolic BP >90 mm Hg.

ANS: A Furosemide will lower the filling pressures and renal perfusion further for the patient with septic shock. The other orders are appropriate. DIF: Cognitive Level: Apply (application) REF: 1640

A client has developed acute respiratory distress syndrome (ARDS). What is the priority diagnosis for the client? a. Impaired gas exchange b. Risk for infection c. Imbalanced nutrition: Less than body requirements d. Impaired skin integrity

ANS: A Impaired gas exchange is the priority nursing diagnosis. A client with ARDS usually requires intubation and mechanical ventilation. The other diagnoses are appropriate but not the priority. ANALYSIS (Made Easy)

The nurse reviews the assessment have a client with a cute mess between your distress syndrome (ARDS). What is the best indicator of improvement in the client? a. Arterial blood gas (ABG) values b. bronchoscopy results c. Increased blood pressure d. Sputum culture and sensitivity results

ANS: A Improved ABG results would indicate that the clients oxygenation status is improved. Hypoxia is the problem and ARDS, so bronchoscopy and sputum culture results may have no bearing on the improvement of ARDS. Increased blood pressure isn't relative to the clients respiratory condition. ANALYSIS (Made Easy)

A 19-year-old client admitted with heat stroke begins to show signs of disseminated intravascular coagulation (DIC). The nurse analyzes the laboratory findings and identifies which finding as most consistent with DIC? a. low platelet count b. elevated fibrinogen levels c. low levels of fibrin degradation products d. reduced prothrombin time (PT)

ANS: A In DIC, platelets and clotting factors are consumed, resulting in microthrombi and excessive bleeding. As clots form, fibrinogen levels decrease and the PT increases. Fibrin degradation products increase as fibrinolysis takes place. ANALYSIS (Made Easy)

The client has been admitted to the burn unit with extensive for thickness burns. What is the nurses priority and implementing the treatment plan for the client? a. Fluid status b. Body image c. Level of pain d. Risk of infection

ANS: A In early burn care, the clients greatest need is fluid resuscitation because of large volume fluid loss through the damage skin. Body image, pain, and infection or important concerns in the nursing care of a burn patient but don't take precedence over food management in the early phase of burn care. ANALYSIS (Made Easy)

A client experienced smoke inhalation and developed pulmonary edema. The nurse auscultates the client's breath sounds and anticipates hearing which of the following? a. crackles b. decreased breath sounds c. inspiratory and expiratory wheezing d. upper airway rhonchi

ANS: A In pulmonary edema, the most frequently hear sounds are crackles. APPLICATION (Made Easy)

A patient has been on the medical floor for 1 week after a vaginal hysterectomy. A urinary catheter was inserted. Complete blood cell count results have revealed escalating white blood cell counts. The patient is transferred to the critical care unit when her condition deteriorates. Septic shock is diagnosed. A pulmonary artery catheter is placed. Which of the following hemodynamic values would you expect to find? a. Cardiac output of 8 L/min b. Right atrial pressure of 17 mm Hg c. Pulmonary artery wedge pressure of 23 mm Hg d. Systemic vascular resistance of 110

ANS: A Increased cardiac output and decreased systemic vascular resistance are classic signs of septic shock.

While caring for a patient who has been admitted with a pulmonary embolism, the nurse notes a change in the patient's oxygen saturation (SpO2) from 94% to 88%. Which action should the nurse take next? a. Increase the oxygen flow rate. b. Suction the patient's oropharynx. c. Instruct the patient to cough and deep breathe. d. Help the patient to sit in a more upright position.

ANS: A Increasing oxygen flow rate will usually improve oxygen saturation in patients with ventilation-perfusion mismatch, as occurs with pulmonary embolism. Because the problem is with perfusion, actions that improve ventilation, such as deep breathing and coughing, sitting upright, and suctioning, are not likely to improve oxygenation. DIF: Cognitive Level: Apply (application) REF: 1656

When admitting a patient with possible respiratory failure with a high PaCO2, which assessment information should be immediately reported to the health care provider? a. The patient is somnolent. b. The patient complains of weakness. c. The patient's blood pressure is 164/98. d. The patient's oxygen saturation is 90%.

ANS: A Increasing somnolence will decrease the patient's respiratory rate and further increase the PaCO2 and respiratory failure. Rapid action is needed to prevent respiratory arrest. An SpO2 of 90%, weakness, and elevated blood pressure all require ongoing monitoring but are not indicators of possible impending respiratory arrest. DIF: Cognitive Level: Apply (application) REF: 1660

A patient was admitted to the critical care unit with acute respiratory failure. The patient has been on a ventilator for 3 days and is being considered for weaning. The ventilator highpressure alarm keeps going off. When you enter the room, the ventilator inoperative alarm sounds. All of the following conditions would set off the highpressure alarm except a. a leak in the patient's ET tube cuff b. a kink in the ET tubing c. coughing d. increased secretions in the patient's airway

ANS: A Low inspiratory pressure alarms will sound because of altered settings, unattached tubing or a leak around the endotracheal tube (ETT), the ETT displaced into the pharynx or esophagus, poor cuff inflation or leak, tracheoesophageal fistula, peak flows that are too low, low tidal volume (Vt), decreased airway resistance resulting from decreased secretions or relief of bronchospasm, increased lung compliance resulting from decreased atelectasis, reduction in pulmonary edema, resolution of ARDS, or a change in position. High pressure alarms will sound because of improper alarm setting; airway obstruction resulting from patient fighting ventilator (holding breath as ventilator delivers Vt); patient circuit collapse; kinked tubing; the ETT in the right mainstem bronchus or against the carina; cuff herniation; increased airway resistance resulting from bronchospasm, airway secretions, plugs, and coughing; water from the humidifier in the ventilator tubing; and decreased lung compliance resulting from tension pneumothorax, change in patient position, acute respiratory distress syndrome, pulmonary edema, atelectasis, pneumonia, or abdominal distention.

The nursing measure that can best enhance large volumes of fluid replacement in hypovolemic shock is a. insertion of a large diameter peripheral intravenous catheter. b. positioning the patient in the Trendelenburg position. c. forcing at least 240 mL of fluid each hour. d. administering intravenous lines under pressure.

ANS: A Measures to facilitate the administration of volume replacement include insertion of largebore peripheral intravenous catheters; rapid administration of prescribed fluids; and positioning the patient with the legs elevated, trunk flat, and head and shoulders above the chest.

A 20-year-old patient arrives in the emergency department (ED) several hours after taking "25 to 30" acetaminophen (Tylenol) tablets. Which action will the nurse plan to take? a. Give N-acetylcysteine (Mucomyst). b. Discuss the use of chelation therapy. c. Start oxygen using a non-rebreather mask. d. Have the patient drink large amounts of water.

ANS: A N-acetylcysteine is the recommended treatment to prevent liver damage after acetaminophen overdose. The other actions might be used for other types of poisoning, but they will not be appropriate for a patient with acetaminophen poisoning. DIF: Cognitive Level: Understand (comprehension) REF: 1689

A patient was admitted to the critical care unit after a left pneumonectomy. The patient is receiving 40% oxygen via a simple facemask. The morning chest radiography study reveals right lower lobe pneumonia. After eating breakfast, the patient suddenly vomits and aspirates. The patient becomes agitated, has decreased level of consciousness, and has an inability to maintain saturation. The nurse expects the next action will include a. placing the patient on a mechanical ventilator. b. change in antibiotics to control infection. c. suctioning and repositioning. d. administering a sedative to control anxiety.

ANS: A Nursing interventions include optimizing oxygenation and ventilation, preventing the spread of infection, providing comfort and emotional support, and maintaining surveillance for complications.

A client who developed a fat embolism receiving 100% FIO2 on a mechanical ventilator and continues to be hypoxic. What is the most important invention? a. Add a positive end-expiratory pressure (PEEP). b. Give Beta adrenergic blockers. c. Give diuretics. d. Increase the FIO2 on the ventilator.

ANS: A PEEP can be added to open up alveoli and keep them open. There's no reason to give the client beta-adrenergic blockers. He may benefit from diuresis, but in the meantime, PEEP should be added to improve oxygenation. The highest amount of oxygen that can be delivered is 100% FIO2. APPLICATION (Made Easy)

With anaphylactic shock, which mechanism results in a decreased cardiac output? a. Peripheral vasodilation b. Increased cardiac output c. Decreased alveolar ventilation d. Fluid retention resulting in congestive heart failure

ANS: A Peripheral vasodilation results in decreased venous return. This decreases intravascular volume and the development of relative hypovolemia. Decreased venous return results in decreased stroke volume and a fall in cardiac output.

Which of the following nursing interventions should be used to optimize oxygenation and ventilation in the patient with acute respiratory failure? a. Provide adequate rest and recovery time between procedures. b. Position the patient with the good lung up. c. Suction the patient every hour. d. Avoid hyperventilating the patient.

ANS: A Providing adequate rest and recovery time between various procedures prevents desaturation and optimizes oxygenation. In acute lung failure, the goal of positioning is to place the least affected area of the patient's lung in the most dependent position. Patients with unilateral lung disease should be positioned with the healthy lung in a dependent position. Hyperventilate the patient before suctioning; suction patients as needed.

Which of the following clinical manifestations is not suggestive of systemic inflammatory response syndrome (SIRS)? a. Temperature of 37.5° C b. Heart rate of 95 beats/min c. Respiratory rate of 24 breath/min d. White blood cell (WBC) count of 15,000 cells/mm2

ANS: A SIRS occurs when two or more of four clinical manifestations are present in the patient at high risk. These manifestations are temperature less than 36° C or greater than 38° C, heart rate greater than 90 beats/min, respiratory rate greater than 20 breaths/min or PaCO less than 32 mm Hg, or WBC greater than 12,000 cells/mm2 or less than 4000 cells/mm2 or greater than 10% immature (band forms).

A firefighter is being treated for smoke inhalationn. He develops severe hypoxia 48 hours later, requiring intubation and mechanical ventilation. The nurse determines that the client is experiencing: a. acute respiratory distress syndrome (ARDS) b. atelectasis c. bronchitis d. pnemonia

ANS: A Severe hypoxia after smoke inhalation is typically related to ARDS. APPLICATION (Made Easy)

When planning the response to the potential use of smallpox as an agent of terrorism, the emergency department (ED) nurse manager will plan to obtain adequate quantities of a. vaccine. b. atropine. c. antibiotics. d. whole blood.

ANS: A Smallpox infection can be prevented or ameliorated by the administration of vaccine given rapidly after exposure. The other interventions would be helpful for other agents of terrorism but not for smallpox. DIF: Cognitive Level: Understand (comprehension) REF: 1690

The charge nurse observes the following actions being taken by a new nurse on the burn unit. Which action by the new nurse would require an intervention by the charge nurse? a. The new nurse uses clean latex gloves when applying antibacterial cream to a burn wound. b. The new nurse obtains burn cultures when the patient has a temperature of 95.2° F (35.1° C). c. The new nurse administers PRN fentanyl (Sublimaze) IV to a patient 5 minutes before a dressing change. d. The new nurse calls the health care provider for a possible insulin order when a nondiabetic patient's serum glucose is elevated.

ANS: A Sterile gloves should be worn when applying medications or dressings to a burn. Hypothermia is an indicator of possible sepsis, and cultures are appropriate. Nondiabetic patients may require insulin because stress and high calorie intake may lead to temporary hyperglycemia. Fentanyl peaks 5 minutes after IV administration, and should be used just before and during dressing changes for pain management. DIF: Cognitive Level: Apply (application) REF: 461

A nurse is caring for a patient with acute respiratory distress syndrome (ARDS) who is receiving mechanical ventilation using synchronized intermittent mandatory ventilation (SIMV). The settings include fraction of inspired oxygen (FIO2) 80%, tidal volume 450, rate 16/minute, and positive end-expiratory pressure (PEEP) 5 cm. Which assessment finding is most important for the nurse to report to the health care provider? a. Oxygen saturation 99% b. Respiratory rate 22 breaths/minute c. Crackles audible at lung bases d. Heart rate 106 beats/minute

ANS: A The FIO2 of 80% increases the risk for oxygen toxicity. Because the patient's O2 saturation is 99%, a decrease in FIO2 is indicated to avoid toxicity. The other patient data would be typical for a patient with ARDS and would not need to be urgently reported to the health care provider. DIF: Cognitive Level: Analyze (analysis) REF: 1669

The nurse is caring for a 33-year-old patient who arrived in the emergency department with acute respiratory distress. Which assessment finding by the nurse requires the most rapid action? a. The patient's PaO2 is 45 mm Hg. b. The patient's PaCO2 is 33 mm Hg. c. The patient's respirations are shallow. d. The patient's respiratory rate is 32 breaths/minute.

ANS: A The PaO2 indicates severe hypoxemia and respiratory failure. Rapid action is needed to prevent further deterioration of the patient. Although the shallow breathing, rapid respiratory rate, and low PaCO2 also need to be addressed, the most urgent problem is the patient's poor oxygenation. DIF: Cognitive Level: Apply (application) REF: 1658 | 1665

During change-of-shift report, the nurse is told that a patient has been admitted with dehydration and hypotension after having vomiting and diarrhea for 4 days. Which finding is most important for the nurse to report to the health care provider? a. New onset of confusion b. Heart rate 112 beats/minute c. Decreased bowel sounds d. Pale, cool, and dry extremities

ANS: A The changes in mental status are indicative that the patient is in the progressive stage of shock and that rapid intervention is needed to prevent further deterioration. The other information is consistent with compensatory shock. DIF: Cognitive Level: Apply (application) REF: 1639

The nurse suspects the client may be developing ARDS. Which assessment data confirm the diagnosis of ARDS? a. Though arterial oxygen when administering high concentration of oxygen. b. The client has dyspnea and tachycardia and is feeling anxious. c. Bilateral breath sounds clear and post oximeter reading is 95%. d. The client has jugular vein distention and frothy sputum.

ANS: A The classic sign of ARDS is decreased arterial oxygen level (PaO2) while administering high levels of oxygen; the oxygen is unable to cross the alveolar membrane. ANALYSIS

A patient with circumferential burns of both legs develops a decrease in dorsalis pedis pulse strength and numbness in the toes. Which action should the nurse take? a. Notify the health care provider. b. Monitor the pulses every 2 hours. c. Elevate both legs above heart level with pillows. d. Encourage the patient to flex and extend the toes on both feet.

ANS: A The decrease in pulse in a patient with circumferential burns indicates decreased circulation to the legs and the need for an escharotomy. Monitoring the pulses is not an adequate response to the decrease in circulation. Elevating the legs or increasing toe movement will not improve the patient's circulation. DIF: Cognitive Level: Apply (application) REF: 458

Which data collected by the nurse caring for a patient who has cardiogenic shock indicate that the patient may be developing multiple organ dysfunction syndrome (MODS)? a. The patient's serum creatinine level is elevated. b. The patient complains of intermittent chest pressure. c. The patient's extremities are cool and pulses are weak. d. The patient has bilateral crackles throughout lung fields.

ANS: A The elevated serum creatinine level indicates that the patient has renal failure as well as heart failure. The crackles, chest pressure, and cool extremities are all consistent with the patient's diagnosis of cardiogenic shock. DIF: Cognitive Level: Apply (application) REF: 1649 | 1633

The finding of normal breath sounds on the right side of the chest and absent breath sounds on the left side of the chest in a newly intubated patient is probably caused by a a. right mainstem intubation. b. left pneumothorax. c. right hemothorax. d. gastric intubation.

ANS: A The finding of normal breath sounds on the right side of the chest and absent breath sounds on the left side of the chest in a newly intubated patient is probably caused by a right mainstem intubation.

A patient is admitted to the emergency department (ED) for shock of unknown etiology. The first action by the nurse should be to a. administer oxygen. b. obtain a 12-lead electrocardiogram (ECG). c. obtain the blood pressure. d. check the level of consciousness.

ANS: A The initial actions of the nurse are focused on the ABCs—airway, breathing, and circulation—and administration of oxygen should be done first. The other actions should be accomplished as rapidly as possible after oxygen administration. DIF: Cognitive Level: Apply (application) REF: 1641

When assessing an older patient admitted to the emergency department (ED) with a broken arm and facial bruises, the nurse observes several additional bruises in various stages of healing. Which statement or question by the nurse is most appropriate? a. "Do you feel safe in your home?" b. "You should not return to your home." c. "Would you like to see a social worker?" d. "I need to report my concerns to the police."

ANS: A The nurse's initial response should be to further assess the patient's situation. Telling the patient not to return home may be an option once further assessment is done. A social worker may be appropriate once further assessment is completed. DIF: Cognitive Level: Apply (application) REF: 1682

The unlicensed assistive personnel (UAP) is bathing the client diagnosed with acute respiratory distress syndrome (ARDS). The bed is in a high position with the opposite side rail in the low position. Which action should the nurse implement? a. Demonstrate the correct technique for giving a bed bath. b. Encourage the UAP to put the bed in the lowest position. c. Instruct the UAP to get another person to help with the bath. d. Provide praise for performing the bath safely for the client and the UAP.

ANS: A The opposite side rail should be elevated so the client will not fall out of bed. Safety is priority, the nurse should demonstrate the proper way to bathe a client in the bed. SYNTHESIS

A nurse is caring for an obese patient with right lower lobe pneumonia. Which position will be best to improve gas exchange? a. On the left side b. On the right side c. In the tripod position d. In the high-Fowler's position

ANS: A The patient should be positioned with the "good" lung in the dependent position to improve the match between ventilation and perfusion. The obese patient's abdomen will limit respiratory excursion when sitting in the high-Fowler's or tripod positions. DIF: Cognitive Level: Apply (application) REF: 1662

The most common presenting signs and symptoms associated with PEs are a. tachycardia and tachypnea. b. hemoptysis and evidence of deep vein thromboses. c. apprehension and dyspnea. d. right ventricular failure and fever

ANS: A The patient with a pulmonary embolism may have any number of presenting signs and symptoms, with the most common being tachycardia and tachypnea. Additional signs and symptoms that may be present include dyspnea, apprehension, increased pulmonic component of the second heart sound (P1), fever, crackles, pleuritic chest pain, cough, evidence of deep vein thrombosis, and hemoptysis. Syncope and hemodynamic instability can occur as a result of right ventricular failure.

A patient with multisystem trauma has been in the ICU for 6 days after sustaining a closed head injury, a right-sided pneumothorax, right rib fractures, a grade IV liver laceration, a pancreatic contusion, and a right acetabular fracture. The patient is still intubated and mechanically ventilated and has a chest tube, Foley catheter, and two abdominal drains. The patient's hemodynamic assessment reveals the following values: BP, 94/66 mm Hg; HR, 118 beats/min; T, 38.7° C; CVP, 5 cm H2O; wedge pressure, 6 mm Hg; cardiac index, 6.1; and systemic vascular resistance, 450 dynes/sec. What is the most likely cause of this hemodynamic picture? a. Septic shock b. Hemorrhagic shock c. Cardiogenic shock d. Neurogenic shock

ANS: A The patient with multiple injuries is at risk for overwhelming infections and sepsis. The source of sepsis in the trauma patient can be invasive therapeutic and diagnostic catheters or wound contamination with exogenous or endogenous bacteria. The source of the septic nidus must be promptly evaluated. Gram stain and cultures of blood, urine, sputum, invasive catheters, and wounds are obtained.

A nurse is caring for a patient with shock of unknown etiology whose hemodynamic monitoring indicates BP 92/54, pulse 64, and an elevated pulmonary artery wedge pressure. Which collaborative intervention ordered by the health care provider should the nurse question? a. Infuse normal saline at 250 mL/hr. b. Keep head of bed elevated to 30 degrees. c. Hold nitroprusside (Nipride) if systolic BP <90 mm Hg. d. Titrate dobutamine (Dobutrex) to keep systolic BP >90 mm Hg.

ANS: A The patient's elevated pulmonary artery wedge pressure indicates volume excess. A saline infusion at 250 mL/hr will exacerbate the volume excess. The other actions are appropriate for the patient. DIF: Cognitive Level: Apply (application) REF: 1633

A triage nurse in a busy emergency department (ED) assesses a patient who complains of 7/10 abdominal pain and states, "I had a temperature of 103.9° F (39.9° C) at home." The nurse's first action should be to a. assess the patient's current vital signs. b. give acetaminophen (Tylenol) per agency protocol. c. ask the patient to provide a clean-catch urine for urinalysis. d. tell the patient that it will 1 to 2 hours before being seen by the doctor.

ANS: A The patient's pain and statement about an elevated temperature indicate that the nurse should obtain vital signs before deciding how rapidly the patient should be seen by the health care provider. A urinalysis may be appropriate, but this would be done after the vital signs are taken. The nurse will not give acetaminophen before confirming a current temperature elevation. DIF: Cognitive Level: Apply (application) REF: 1675-1676

The nurse reviews the electronic medical record for a patient scheduled for a total hip replacement. Which assessment data shown in the accompanying figure increase the patient's risk for respiratory complications after surgery? a. Albumin level and recent weight loss b. Mild confusion and recent weight loss c. Age and recent arthroscopic procedure. d. Anemia and recent arthroscopic procedure

ANS: A The patient's recent weight loss and low protein stores indicate possible muscle weakness, which make it more difficult for an older patient to recover from the effects of general anesthesia and immobility associated with the hip surgery. The other information will also be noted by the nurse but does not place the patient at higher risk for respiratory failure. DIF: Cognitive Level: Analyze (analysis) REF: 1665

The following interventions are part of the emergency department (ED) protocol for a patient who has been admitted with multiple bee stings to the hands. Which action should the nurse take first? a. Remove the patient's rings. b. Apply ice packs to both hands. c. Apply calamine lotion to any itching areas. d. Give diphenhydramine (Benadryl) 50 mg PO.

ANS: A The patient's rings should be removed first because it might not be possible to remove them if swelling develops. The other orders should also be implemented as rapidly as possible after the nurse has removed the jewelry. DIF: Cognitive Level: Apply (application) REF: 1687

The nurse caring for a patient admitted with burns over 30% of the body surface assesses that urine output has dramatically increased. Which action by the nurse would best ensure adequate kidney function? a. Continue to monitor the urine output. b. Monitor for increased white blood cells (WBCs). c. Assess that blisters and edema have subsided. d. Prepare the patient for discharge from the burn unit.

ANS: A The patient's urine output indicates that the patient is entering the acute phase of the burn injury and moving on from the emergent stage. At the end of the emergent phase, capillary permeability normalizes and the patient begins to diurese large amounts of urine with a low specific gravity. Although this may occur at about 48 hours, it may be longer in some patients. Blisters and edema begin to resolve, but this process requires more time. White blood cells may increase or decrease, based on the patient's immune status and any infectious processes. The WBC count does not indicate kidney function. The patient will likely remain in the burn unit during the acute stage of burn injury. DIF: Cognitive Level: Understand (comprehension) REF: 464

A patient was admitted to the critical care unit with gram negative sepsis 5 days ago. Today there is continual oozing from his intravenous sites, and ecchymosis of the skin is noted beneath his automatic blood pressure cuf . On his laboratory work, his platelets are normal, and his international normalized ratio is elevated. The primary treatment goal for this patient is to a. maintain adequate organ perfusion. b. suppress antibody response that is destroying platelets. c. treat lifethreatening metabolic disturbances. d. begin hypothermic therapy to prevent cerebral hemorrhage.

ANS: A The primary intervention in disseminated intravascular coagulation (DIC) is prevention. Being aware of the conditions that commonly contribute to the development of DIC and treating them vigorously and without delay provide the best defense against this devastating condition. After DIC is identified, maintaining organ perfusion and slowing consumption of coagulation factors are paramount to achieving a favorable outcome.

An unresponsive 79-year-old is admitted to the emergency department (ED) during a summer heat wave. The patient's core temperature is 105.4° F (40.8° C), blood pressure (BP) 88/50, and pulse 112. The nurse initiallywill plan to a. apply wet sheets and a fan to the patient. b. provide O2 at 6 L/min with a nasal cannula. c. start lactated Ringer's solution at 1000 mL/hr. d. give acetaminophen (Tylenol) rectal suppository.

ANS: A The priority intervention is to cool the patient. Antipyretics are not effective in decreasing temperature in heat stroke, and 100% oxygen should be given, which requires a high flow rate through a non-rebreather mask. An older patient would be at risk for developing complications such as pulmonary edema if given fluids at 1000 mL/hr. DIF: Cognitive Level: Apply (application) REF: 1683

A patient with acute respiratory distress syndrome (ARDS) is placed in the prone position. When prone positioning is used, which information obtained by the nurse indicates that the positioning is effective? a. The patient's PaO2 is 89 mm Hg, and the SaO2 is 91%. b. Endotracheal suctioning results in clear mucous return. c. Sputum and blood cultures show no growth after 48 hours. d. The skin on the patient's back is intact and without redness.

ANS: A The purpose of prone positioning is to improve the patient's oxygenation as indicated by the PaO2 and SaO2. The other information will be collected but does not indicate whether prone positioning has been effective. DIF: Cognitive Level: Apply (application) REF: 1670

Following an earthquake, patients are triaged by emergency medical personnel and are transported to the emergency department (ED). Which patient will the nurse need to assess first? a. A patient with a red tag b. A patient with a blue tag c. A patient with a black tag d. A patient with a yellow tag

ANS: A The red tag indicates a patient with a life-threatening injury requiring rapid treatment. The other tags indicate patients with less urgent injuries or those who are likely to die. DIF: Cognitive Level: Remember (knowledge) REF: 1692

The most common site for sepsis and septic shock is a. the respiratory system. b. the gastrointestinal system. c. the genitourinary system. d. the circulatory system.

ANS: A The respiratory system is the most common site of infection producing severe sepsis and septic shock followed by the genitourinary and gastrointestinal systems.

The client admitted with full-thickness burns may be developing DIC. Which signs and symptoms would support the diagnosis of DIC? a. Oozing blood from the IV catheter site. b. Suddent onset of chest pain and frothy sputum. c. Foul-smelling, concentrated urine. d. A reddened, inflamed central line catheter site.

ANS: A The signs/symptoms of DIC result from clotting and bleeding, ranging from oozing blood to bleeding from every body orifice and into the tissures. ANALYSIS

A patient has been on the medical floor for 1 week after a vaginal hysterectomy. A urinary catheter was inserted. Complete blood cell count results have revealed escalating white blood cell counts. The patient is transferred to the critical care unit when her condition deteriorates. Septic shock is diagnosed. Which of the following is the pathophysiologic mechanism that results in septic shock? a. Bacterial toxins lead to vasodilation. b. Increased white blood cells are released to fight invading bacteria. c. Microorganisms invade organs such as the kidneys and heart. d. An increase of white blood cells leads to decreased red blood cell production and anemia.

ANS: A The syndrome encompassing severe sepsis and septic shock is a complex systemic response that is initiated when a microorganism enters the body and stimulates the inflammatory or immune system. Shed protein fragments and the release of toxins and other substances from the microorganism activate the plasma enzyme cascades (complement, kinin and kallikrein, coagulation, and fibrinolytic factors), as well as platelets, neutrophils, monocytes, and macrophages.

The healthcare provider ordered STAT arterial blood gases (ABGs) for the client diagnosed with ARDS. The ABG results are pH 7.38, PaO2 92, PaCO2 38, HCO3 24. Which action should the nurse implement? a. Continue to monitor the client without taking any action. b. Encourage the client to take deep breath's and cough. c. Administer one (1) ampule of sodium bicarbonate IVP. d. Notify the respiratory therapist of the ABG results.

ANS: A These arterial blood gases are within normal limits, and, therefore, the nurse should not take any action except to continue to monitor the client. APPLICATION

The charge nurse in the intensive care unit is making client assignments. Which client should the charge nurse assign to the graduate nurse who has just finished the three (3) -month orientation? a. the client with an abdominal peritoneal resection who has a colostomy. b. the client diagnosed with pneumonia who has acute respiratory distress syndrome. c. the clent with a head injury developing disseminated intravascular coagulation. d. the client admitted with a gunshot wound who has an H&H of 7 and 22.

ANS: A This is a major surgery but has a predictable course with no complications identified in the stem, and a colostomy is expected with this type of surgery. The graduate nurse could be assigned this patient. SYNTHESIS

Which patient should the nurse assess first? a. A patient with smoke inhalation who has wheezes and altered mental status b. A patient with full-thickness leg burns who has a dressing change scheduled c. A patient with abdominal burns who is complaining of level 8 (0 to 10 scale) pain d. A patient with 40% total body surface area (TBSA) burns who is receiving IV fluids at 500 mL/hour

ANS: A This patient has evidence of lower airway injury and hypoxemia and should be assessed immediately to determine the need for oxygen or intubation. The other patients should also be assessed as rapidly as possible, but they do not have evidence of life-threatening complications. DIF: Cognitive Level: Apply (application) REF: 452

The client diagnosed with ARDS is on a ventilator and the high alarm indicates an increase in the peak airway pressure. Which intervention should the nurse implement first? a. Check the tubing for any kinks. b. Section the airway for secretions. c. Assess the lip line of the ET tube. d. The date the client with a muscle relaxant.

ANS: A When peak airway pressure is increased, the nurse should implement the intervention least invasive for the client. This alarm goes off with a plugged airway, "bucking" in the ventilator, decreasing lung compliance, kinked tubing, or pneumothorax. SYNTHESIS

After the return of spontaneous circulation following the resuscitation of a patient who had a cardiac arrest, therapeutic hypothermia is ordered. Which action will the nurse include in the plan of care? a. Apply external cooling device. b. Check mental status every 15 minutes. c. Avoid the use of sedative medications. d. Rewarm if temperature is <91° F (32.8° C).

ANS: A When therapeutic hypothermia is used postresuscitation, external cooling devices or cold normal saline infusions are used to rapidly lower body temperature to 89.6° F to 93.2° F (32° C to 34° C). Because hypothermia will decrease brain activity, assessing mental status every 15 minutes is not needed at this stage. Sedative medications are administered during therapeutic hypothermia. DIF: Cognitive Level: Apply (application) REF: 1681

The assistcontrol mode of ventilation functions in which of the following manners? a. It delivers gas at preset volume, at a set rate, and in response to the patient's inspiratory efforts. b. It delivers gas at a preset volume, allowing the patient to breathe spontaneously at his or her own volume. c. It applies positive pressure during both ventilator breaths and spontaneous breaths. d. It delivers gas at preset rate and tidal volume regardless of the patient's inspiratory efforts.

ANS: A Whereas a breath that is initiated by the patient is known as a patient triggered or patientassisted breath, a breath that is initiated by the ventilator is known as a machinetriggered or machinecontrolled breath. Atimetriggered breath is a machinecontrolled breath that is initiated by the ventilator after a preset length of time has elapsed. It is controlled by the rate setting on the ventilator (e.g., a rate of 10 breaths/min yields 1 breath every 6 seconds). Flowtriggered and pressuretriggered breaths are patientassisted breaths that are initiated by decreased flow or pressure, respectively, within the breathing circuit.

Which of the following oxygen administration devices can deliver oxygen concentrations of 90%? a. Nonrebreathing mask b. Nasal cannula c. Partial rebreathing mask d. Simple mask

ANS: A With an FiO2 of 55% to 70%, a nonrebreathing mask with a tight seal over the face can deliver 90% to 100% oxygen. It is used in emergencies and shortterm therapy requiring moderate to high FiO2.

An employee spills industrial acids on both arms and legs at work. What is the priority action that the occupational health nurse at the facility should take? a. Remove nonadherent clothing and watch. b. Apply an alkaline solution to the affected area. c. Place cool compresses on the area of exposure. d. Cover the affected area with dry, sterile dressings.

ANS: A With chemical burns, the initial action is to remove the chemical from contact with the skin as quickly as possible. Remove nonadherent clothing, shoes, watches, jewelry, glasses, or contact lenses (if face was exposed). Flush chemical from wound and surrounding area with copious amounts of saline solution or water. Covering the affected area or placing cool compresses on the area will leave the chemical in contact with the skin. Application of an alkaline solution is not recommended. DIF: Cognitive Level: Apply (application) REF: 455 | 457

The nurse is preparing a plan of care for a client requiring a dressing change. What is the most important nursing intervention? a. Write the order in the client's care plan. b. Put the sign above the head of the clients bed. c. tell the nurse about the treatment in the report. d. Document the dressing change in the narrative note.

ANS: A Writing the order in the clients care plan notifies everyone of the treatment. Posting a sign above the head of the bed is a good reminder but doesn't ensure that the treatment will be performed. Verbally reporting to the nurse on the upcoming shift doesn't end sure the dressing change will be done. Although the intervention should be documented in the narrative note, this doesn't guarantee that the next nurse will do the treatment. APPLICATION (Made Easy)

Which of the following levels would be classified as a lowflow system of oxygen administration? a. O2 via nasal cannula at 4 L/min b. O2 via nasal catheter at a FiO2 range of 60% to 75% c. O2 via transtracheal catheter at 10 L/min d. O2 via simple mask at 12 L/min.

ANS: A A low flow oxygen delivery system provides supplemental oxygen directly into the patient's airway at a flow of 8 L/min or less. Because this flow is insufficient to meet the patient's inspiratory volume requirements, it results in a variable FiO2 as the supplemental oxygen is mixed with room air. Nasal catheter FiO2 range is 22% to 45%. Oxygen flow through a transtracheal catheter is 0.25 to 4 L/min. A simple mask is a reservoir delivery system.

Which instruction is priority for the nurse to discuss with the client diagnosed with ARDS who is being discharged from the hospital? a. Avoid smoking and exposure to smoke. b. Do not receive flu or pneumonia vaccines. c. Avoid any type of alcohol intake. d. It will take about one month to recuperate.

ANS: A Non-smoking is vital to prevent further lung damage. SYNTHESIS

When preparing to cool a patient who is to begin therapeutic hypothermia, which intervention will the nurse plan to do (select all that apply)? a. Assist with endotracheal intubation. b. Insert an indwelling urinary catheter. c. Begin continuous cardiac monitoring. d. Obtain an order to restrain the patient. e. Prepare to give sympathomimetic drugs.

ANS: A, B, C Cooling can produce dysrhythmias, so the patient's heart rhythm should be continuously monitored and dysrhythmias treated if necessary. Bladder catheterization and endotracheal intubation are needed during cooling. Sympathomimetic drugs tend to stimulate the heart and increase the risk for fatal dysrhythmias such as ventricular fibrillation. Patients receiving therapeutic hypothermia are comatose or do not follow commands so restraints are not indicated. DIF: Cognitive Level: Apply (application) REF: 1681

Which of the following should be used when suctioning a mechanically ventilated patient? (Select all that apply.) a. Three hyperoxygenation breaths (breaths at 100% FiO ) b. Hyperinflation (breaths at 150% tidal volume) c. Limit the number of passes to three. d. Instill 5 to 10 mL of normal saline to facilitate secretion removal. e. Use intermittent suction to avoid damaging tracheal tissue.

ANS: A, B, C Hyperoxygenation, hyperinflation, and limiting the number of passes help avoid desaturation. There is no evidence to suggest that intermittent suction reduces damage, and saline instillation can actually increase the risk for infection.

The key to treatment of septic shock is finding the cause of the infection. Which of the following cultures are obtained before antibiotic therapy is initiated? (Select all that apply.) a. Blood cultures x 2 b. Wound cultures c. Urine cultures d. Sputum cultures e. CBC with differential

ANS: A, B, C, D A key measure in the treatment of septic shock is finding and eradicating the cause of the infection. At least two blood cultures plus urine, sputum, and wound cultures should be obtained to find the location of the infection before antibiotic therapy is initiated. Antibiotic therapy should be started within 1 hour of recognition of severe sepsis without delay for cultures.

Weaning methods that are used in combination with each other include(Select all that apply.) a. SIMV with CPAP. b. SIMV with PSV. c. CPAP with PSV. d. Tpiece and PSV. e. PEEP with CPAP.

ANS: A, B, C, D A variety of weaning methods are available, but no one method has consistently proven to be superior to the others. These methods include Ttube (Tpiece), continuous positive airway pressure (CPAP), pressure support ventilation (PSV), and synchronized intermittent mandatory ventilation (SIMV). One recent multicenter study lends evidence to support the use of PSV for weaning over Ttube or SIMV weaning. Often these weaning methods are used in combination with each other, such as SIMV with PSV, CPAP with PSV, or SIMV with CPAP.

Psychologic factors that may contribute to longterm mechanical ventilation dependence include (Select all that apply.) a. fear. b. delirium. c. lack of confidence in the ability to breathe. d. depression. e. trust in the staff so the patient displays a lack of effort.

ANS: A, B, C, D Psychologic factors contributing to longterm mechanical ventilation dependence include a loss of breathing pattern control (anxiety, fear, dyspnea, pain, ventilator asynchrony, lack of confidence in ability to breathe), lack of motivation and confidence (inadequate trust in staff, depersonalization, hopelessness, powerlessness, depression, inadequate communication), and delirium (sensory overload, sensory deprivation, sleep deprivation, pain medications).

Complications of ETT tubes include (Select all that apply.) a. tracheoesophageal fistula. b. cricoid abscess. c. tracheal stenosis. d. tube obstruction. e. tube displacements.

ANS: A, B, C, D, E Complications of endotracheal tubes include tube obstruction, tube displacement, sinusitis and nasal injury, tracheoesophageal fistula, mucosal lesions, laryngeal or tracheal stenosis, and cricoid abscess.

Which preventive actions by the nurse will help limit the development of systemic inflammatory response syndrome (SIRS) in patients admitted to the hospital (select all that apply)? a. Use aseptic technique when caring for invasive lines or devices. b. Ambulate postoperative patients as soon as possible after surgery. c. Remove indwelling urinary catheters as soon as possible after surgery. d. Advocate for parenteral nutrition for patients who cannot take oral feedings. e. Administer prescribed antibiotics within 1 hour for patients with possible sepsis.

ANS: A, B, C, E Because sepsis is the most frequent etiology for SIRS, measures to avoid infection such as removing indwelling urinary catheters as soon as possible, use of aseptic technique, and early ambulation should be included in the plan of care. Adequate nutrition is important in preventing SIRS. Enteral, rather than parenteral, nutrition is preferred when patients are unable to take oral feedings because enteral nutrition helps maintain the integrity of the intestine, thus decreasing infection risk. Antibiotics should be administered within 1 hour after being prescribed to decrease the risk of sepsis progressing to SIRS. DIF: Cognitive Level: Analyze (analysis) REF: 1649

Medical management of patient with status asthmaticus that supports oxygenation and ventilation include (Select all that apply.) a. oxygen therapy. b. bronchodilators. c. corticosteroids. d. antibiotics. e. intubation and mechanical ventilation.

ANS: A, B, C, E Medical management of a patient with status asthmaticus is directed toward supporting oxygenation and ventilation. Bronchodilators, corticosteroids, oxygen therapy, and intubation and mechanical ventilation are the mainstays of therapy.

Which of the following historical findings would indicate a high risk for latex allergy? (Select all that apply.) a. Allergic reaction to anesthetics b. Eczema of the hands c. Congenital urologic disorder d. Asthma e. Health care worker

ANS: A, B, C, E Prevention of anaphylactic shock is one of the primary responsibilities of nurses in critical care areas. Preventive measures include the identification of patients at risk and cautious assessment of each patient's response to the administration of medications, blood, and blood products. A complete and accurate history of each patient's allergies is an essential component of preventive nursing care. In addition to a list of the allergies, a detailed description of the type of response for each one should be obtained.

A patient with suspected neurogenic shock after a diving accident has arrived in the emergency department. A cervical collar is in place. Which actions should the nurse take (select all that apply)? a. Prepare to administer atropine IV. b. Obtain baseline body temperature. c. Infuse large volumes of lactated Ringer's solution. d. Provide high-flow oxygen (100%) by non-rebreather mask. e. Prepare for emergent intubation and mechanical ventilation.

ANS: A, B, D, E All of the actions are appropriate except to give large volumes of lactated Ringer's solution. The patient with neurogenic shock usually has a normal blood volume, and it is important not to volume overload the patient. In addition, lactated Ringer's solution is used cautiously in all shock situations because the failing liver cannot convert lactate to bicarbonate. DIF: Cognitive Level: Apply (application) REF: 1646

Nursing management of the patient with acute lung failure includes which of the following interventions (Select all that apply.) a. Positioning the patient with the least affected side down b. Providing adequate rest between treatments c. Performing percussion and postural drainage every 4 hours d. Controlling fever e. Pharmaceutical medications to control anxiety

ANS: A, B, D, E The goal of positioning is to place the least affected area of the patient's lung in the most dependent position. Patients with unilateral lung disease should be positioned with the healthy lung in a dependent position. Patients with diffuse lung disease may benefit from being positioned with the right lung down because it is larger and more vascular than the left lung. For patients with alveolar hypoventilation, the goal of positioning is to facilitate ventilation. These patients benefit from nonrecumbent positions such as sitting or a semierect position. In addition, semirecumbency has been shown to decrease the risk of aspiration and inhibit the development of hospitalassociated pneumonia. Frequent repositioning (at least every 2 hours) is beneficial in optimizing the patient's ventilatory pattern and ventilation/perfusion matching. These include performing procedures only as needed, hyperoxygenating the patient before suctioning, providing adequate rest and recovery time between various procedures, and minimizing oxygen consumption. Interventions to minimize oxygen consumption include limiting the patient's physical activity, administering sedation to control anxiety, and providing measures to control fever.

Which of the following statements are true regarding rotational therapy? (Select all that apply.) a. Continuous lateral rotation therapy (CLRT) can be effective for improving oxygenation if used for at least 18 hours/day. b. Kinetic therapy can decrease the incidence of ventilator acquired pneumonia in neurologic and post operative patients. c. Use of rotational therapy eliminates the need for other pressure ulcer prevention strategies. d. CLRT helps avoid hemodynamic instability secondary to the continuous, gentle turning of the patient. e. CLRT has minimal pulmonary benefits for critically ill patients.

ANS: A, B, E Studies have found that to achieve benefits with CLRT or kinetic therapy, rotation must be aggressive, and the patient must be at least 40 degrees per side, with a total arc of at least 80 degrees for at least 18 hours a day. Kinetic therapy has been shown to decrease the incidence of ventilatoracquired pneumonia, particularly in neurologic and postoperative patients. Complications of the procedure include dislodgment or obstruction of tubes, drains, and lines; hemodynamic instability; and pressure ulcers. Lateral rotation does not replace manual repositioning to prevent pressure ulcers. CLRT has been shown to be of minimal pulmonary benefit for the critically ill patients.

Identify the clinical manifestations associated with oxygen toxicity.(Select all that apply.) a. Substernal chest pain that increases with deep breathing b. Moist cough and tracheal irritation c. Pleuritic pain occurring on inhalation, followed by dyspnea d. Increasing CO2 e. Sore throat and eye and ear discomfort

ANS: A, C, E A number of clinical manifestations are associated with oxygen toxicity. The first symptom is substernal chest pain that is exacerbated by deep breathing. A dry cough and tracheal irritation follow. Eventually, definite pleuritic pain occurs on inhalation followed by dyspnea. Upper airway changes may include a sensation of nasal stuffiness, sore throat, and eye and ear discomforts.

Evidence based guidelines for the treatment of septic shock include which of the following? (Select all that apply.) a. Fluid resuscitation to maintain central venous pressure at 8 mm Hg or greater b. Lowdose dopamine for renal protection c. Highdose corticosteroids d. Administration of activated protein C e. Achieve central venous oxygen saturation of 70% or more

ANS: A, D, E There is no evidence to support the use of dopamine; lowdose steroids are part of the sepsis management bundle.

Use of oxygen therapy in the patient who is hypercapnic may result in a. oxygen toxicity. b. absorption atelectasis. c. carbon dioxide retention. d. pneumothorax.

ANS: C Deoxygenated hemoglobin carries more CO2 compared with oxygenated hemoglobin. Administration of oxygen increases the proportion of oxygenated hemoglobin, which causes increased release of CO2 at the lung level. Because of the risk of CO2 accumulation, all patients who are chronically hypercapnic require careful low flow oxygen administration.

The nurse is caring for the client diagnosed with ARDS. Which interventions should the nurse implement? Select all that apply. a. Assess the clients level of consciousness. b. Monitor urine output every shift. c. Turn the client every two hours. d. Maintain intravenous fluids as ordered. e. Please the clients in the Fowlers position.

ANS: ACDE altered level of consciousness is the earliest sign of hypoxemia. The client is at risk for complications of immobility; therefore, the nurse should turn the client at least every two hours to prevent pressure ulcers. The client is at risk for fluid volume overload, so the nurse should monitor and maintain the fluid intake. Fowlers position facilitates long expansion and reduces the workload of breathing. APPLICATION

A client with extensive Burns has a new donor site. What is the most important intervention by the nurse? a. Make the site dependent. b. Avoid pressure on the site. c. Keep the site fully covered. d. Allow ventilation of the site.

ANS: B A universal concern in the care of donor sites for burn care is to keep the site away from sources of pressure. Placing the site in a position of dependents isn't a justified aspect of donor site care. Ventilation of the site and keeping the site fully covered our practices in some institutions but aren't hallmarks of donor site care. APPLICATION (Made Easy)

The most common cause of disseminated intravascular coagulation (DIC) is a. sepsis caused by gram positive organisms. b. sepsis caused by gram negative organisms. c. sickle cell anemia. d. burns.

ANS: B Although all of these answers can cause DIC, sepsis, particularly that caused by gram negative organisms, can be identified as the culprit in as many as 20% of cases, making it the most common cause of DIC.

Which finding is the best indicator that the fluid resuscitation for a patient with hypovolemic shock has been effective? a. Hemoglobin is within normal limits. b. Urine output is 60 mL over the last hour. c. Central venous pressure (CVP) is normal. d. Mean arterial pressure (MAP) is 72 mm Hg.

ANS: B Assessment of end organ perfusion, such as an adequate urine output, is the best indicator that fluid resuscitation has been successful. The hemoglobin level, CVP, and MAP are useful in determining the effects of fluid administration, but they are not as useful as data indicating good organ perfusion. DIF: Cognitive Level: Apply (application) REF: 1642

The nurse is caring for a patient who has septic shock. Which assessment finding is most important for the nurse to report to the health care provider? a. Blood pressure (BP) 92/56 mm Hg b. Skin cool and clammy c. Oxygen saturation 92% d. Heart rate 118 beats/minute

ANS: B Because patients in the early stage of septic shock have warm and dry skin, the patient's cool and clammy skin indicates that shock is progressing. The other information will also be reported, but does not indicate deterioration of the patient's status. DIF: Cognitive Level: Apply (application) REF: 1638

A patient admitted with acute respiratory failure has a nursing diagnosis of ineffective airway clearance related to thick, secretions. Which action is a priorityfor the nurse to include in the plan of care? a. Encourage use of the incentive spirometer. b. Offer the patient fluids at frequent intervals. c. Teach the patient the importance of ambulation. d. Titrate oxygen level to keep O2 saturation >93%.

ANS: B Because the reason for the poor airway clearance is the thick secretions, the best action will be to encourage the patient to improve oral fluid intake. Patients should be instructed to use the incentive spirometer on a regular basis (e.g., every hour) in order to facilitate the clearance of the secretions. The other actions may also be helpful in improving the patient's gas exchange, but they do not address the thick secretions that are causing the poor airway clearance. DIF: Cognitive Level: Apply (application) REF: 1662

A patient with acute respiratory failure may require a bronchodilator if which of the following occurs? a. Excessive secretions b. Bronchospasms c. Thick secretions d. Fighting the ventilator

ANS: B Bronchodilators aid in smooth muscle relaxation and are of particular benefit to patients with airflow limitations. Mucolytics and expectorants are no longer used because they have been found to be of no benefit in this patient population.

Which statement by the nurse when explaining the purpose of positive end-expiratory pressure (PEEP) to the family members of a patient with ARDS is accurate? a. "PEEP will push more air into the lungs during inhalation." b. "PEEP prevents the lung air sacs from collapsing during exhalation." c. "PEEP will prevent lung damage while the patient is on the ventilator." d. "PEEP allows the breathing machine to deliver 100% oxygen to the lungs."

ANS: B By preventing alveolar collapse during expiration, PEEP improves gas exchange and oxygenation. PEEP will not prevent lung damage (e.g., fibrotic changes that occur with ARDS), push more air into the lungs, or change the fraction of inspired oxygen (FIO2) delivered to the patient. DIF: Cognitive Level: Understand (comprehension) REF: 1669

Which nursing action is a priority for a patient who has suffered a burn injury while working on an electrical power line? a. Obtain the blood pressure. b. Stabilize the cervical spine. c. Assess for the contact points. d. Check alertness and orientation.

ANS: B Cervical spine injuries are commonly associated with electrical burns. Therefore stabilization of the cervical spine takes precedence after airway management. The other actions are also included in the emergent care after electrical burns, but the most important action is to avoid spinal cord injury. DIF: Cognitive Level: Apply (application) REF: 452 | 456

A 19-year-old client presents with a second-degree sunburn on her face and both arms. What is the initial intervention by the nurse? a. Administer analgesic medication as ordered. b. Apply cold, moist towels to the Burns. c. Apply sterile, dry towels to the Burns. d. Apply vitamin a, D, and E ointment to the Burns.

ANS: B Cold, moist towels help stop the burning process. Analgesic should be administered as ordered after the burning process has been controlled. Dry towels would retain the heat and are used. Ointments are applied during the healing phase but not initially. APPLICATION (Made Easy)

A patient was admitted to the critical care unit with gramnegative sepsis 5 days ago. Today there is continual oozing from his intravenous sites, and ecchymosis of the skin is noted beneath his automatic blood pressure cuf . On his laboratory work, his platelets are normal, and his international normalized ratio is elevated. What other laboratory value would be most valuable in definitively diagnosing the patient's condition? a. Fibrin split products b. DDimer level c. Bleeding time d. White blood cell count

ANS: B DDimers are exclusively indicative of clot degradation and assist in determining the degree of coagulopathy.

A patient is admitted to the intensive care unit after she develops disseminated intravascular coagulation (DIC) after a vaginal delivery. DIC is known to occur in patients with retained placental fragments. What is the result of DIC? a. Hypersensitive response to an antigen, resulting in anaphylaxis b. Depletion of clotting factors and excessive fibrinolysis, resulting in simultaneous microvascular clotting and hemorrhage c. Vasodilatation, resulting in hypotension d. Septic shock, resulting in vasodilation and decreased perfusion.

ANS: B DIC results simultaneously in microvascular clotting and hemorrhage in organ systems, leading to thrombosis and fibrinolysis in life threatening proportions. Clotting factor derangement leads to further inflammation and further thrombosis. Microvascular damage leads to further organ injury. Cell injury and damage to the endothelium activate the intrinsic or extrinsic coagulation pathways.

The client has just arrived at the emergency department after sustaining a major burn injury. During the first 8 hours after the injury, the nurse will assess the client for which of the following? a. Hyponatremia and hypokalemia b. Hyponatremia and hyperkalemia c. Hypernatremia and hypokalemia d. Hypernatremia and hyperkalemia

ANS: B During the first 48 hours after a burn, capillary permeability increases, allowing fluids to shift from the plasma to the interstitial spaces. This fluid is high in sodium, causing a decrease in serum sodium levels. Potassium also leaks from the cells into the plasma, causing hyperkalemia. ANALYSIS (Made Easy)

A patient has been on the medical floor for 1 week after a vaginal hysterectomy. A urinary catheter was inserted. Complete blood cell count results have revealed escalating white blood cell counts. The patient is transferred to the critical care unit when her condition deteriorates. Septic shock is diagnosed. The medical management of the patient's condition is aimed toward a. limiting fluids to minimize the possibility of congestive heart failure. b. finding and eradicating the cause of infection. c. discontinuing invasive monitoring as a possible cause of sepsis. d. administering vasodilator substances to increase blood flow to vital organs.

ANS: B Effective treatment of severe sepsis and septic shock depends on timely recognition. The diagnosis of severe sepsis is based on the identification of three conditions: known or suspected infection, two or more of the clinical indications of the systemic inflammatory response, and evidence of at least one organ dysfunction. Clinical indications of systemic inflammatory response and sepsis were included in the original American College of Chest Physicians/Society of Critical Care Medicine consensus definitions.

A patient was admitted to the critical care unit with acute respiratory failure. The patient has been on a ventilator for 3 days and is being considered for weaning. The ventilator highpressure alarm keeps going off. When you enter the room, the ventilator inoperative alarm sounds. What is the primary action the nurse would take? a. Troubleshoot the ventilator until the problem is found. b. Take the patient off the ventilator and manually ventilate her. c. Call the respiratory therapist for help. d. Silence the ventilator alarms until the problem is resolved.

ANS: B Ensure emergency equipment is at bedside at all times (e.g., manual resuscitation bag connected to oxygen, masks, suction equipment or supplies), including preparations for power failures. If the ventilator malfunctions, the patient should be removed from the ventilator and ventilated manually with a manual resuscitation bag.

The following interventions are ordered by the health care provider for a patient who has respiratory distress and syncope after eating strawberries. Which will the nurse complete first? a. Start a normal saline infusion. b. Give epinephrine (Adrenalin). c. Start continuous ECG monitoring. d. Give diphenhydramine (Benadryl).

ANS: B Epinephrine rapidly causes peripheral vasoconstriction, dilates the bronchi, and blocks the effects of histamine and reverses the vasodilation, bronchoconstriction, and histamine release that cause the symptoms of anaphylaxis. The other interventions are also appropriate but would not be the first ones completed. DIF: Cognitive Level: Apply (application) REF: 1645

An intubated client with full-thickness, circumferential burns to the chest is experiencing pressure from edema that is inhibiting chest wall expansion. The nurse anticipates that which of the following is the priority intervention for the client? a. Cricothyrotomy b. Escharotomy c. Needle thoracentesis d. Insertion of a chest tube

ANS: B Escharotomy is a surgical incision used to relieve pressure from edema. It's needed with circumferential burns that prevent chest expansion or circulatory compromise. ANALYSIS (Made Easy)

During the primary assessment of a victim of a motor vehicle collision, the nurse determines that the patient is breathing and has an unobstructed airway. Which action should the nurse take next? a. Palpate extremities for bilateral pulses. b. Observe the patient's respiratory effort. c. Check the patient's level of consciousness. d. Examine the patient for any external bleeding.

ANS: B Even with a patent airway, patients can have other problems that compromise ventilation, so the next action is to assess the patient's breathing. The other actions are also part of the initial survey but assessment of breathing should be done immediately after assessing for airway patency. DIF: Cognitive Level: Apply (application) REF: 1676

A patient who has experienced blunt abdominal trauma during a motor vehicle collision is complaining of increasing abdominal pain. The nurse will plan to teach the patient about the purpose of a. peritoneal lavage. b. abdominal ultrasonography. c. nasogastric (NG) tube placement. d. magnetic resonance imaging (MRI).

ANS: B For patients who are at risk for intraabdominal bleeding, focused abdominal ultrasonography is the preferred method to assess for intraperitoneal bleeding. An MRI would not be used. Peritoneal lavage is an alternative, but it is more invasive. An NG tube would not be helpful in diagnosis of intraabdominal bleeding. DIF: Cognitive Level: Apply (application) REF: 1678

A patient with acute respiratory distress syndrome (ARDS) and acute kidney injury has the following medications ordered. Which medication should the nurse discuss with the health care provider before giving? a. Pantoprazole (Protonix) 40 mg IV b. Gentamicin (Garamycin) 60 mg IV c. Sucralfate (Carafate) 1 g per nasogastric tube d. Methylprednisolone (Solu-Medrol) 60 mg IV

ANS: B Gentamicin, which is one of the aminoglycoside antibiotics, is potentially nephrotoxic, and the nurse should clarify the drug and dosage with the health care provider before administration. The other medications are appropriate for the patient with ARDS. DIF: Cognitive Level: Apply (application) REF: 1669

A patient was taken to surgery for a left lung resection earlier today. The patient has been in the postoperative care unit for 30 minutes. When you are completing your assessment, you notice that the chest tube has drained 125 cc of red fluid in the past 30 minutes. The nurse contacts the physician and suspects that the patient has developed a. pulmonary edema. b. hemorrhage. c. acute lung failure. d. bronchopleural fistula.

ANS: B Hemorrhage is an early, lifethreatening complication that can occur after a lung resection. It can result from bronchial or intercostal artery bleeding or disruption of a suture or clip around a pulmonary vessel. Excessive chest tube drainage can signal excessive bleeding. During the immediate postoperative period, chest tube drainage should be measured every 15 minutes; this frequency should be decreased as the patient stabilizes. If chest tube loss is greater than 100 mL/hr, fresh blood is noted, or a sudden increase in drainage occurs, hemorrhage should be suspected.

Profound weight loss in patients with SIRS or MODS is the result of a. hypometabolism. b. hypermetabolism. c. hyperglycemia. d. intolerance to enteral feedings.

ANS: B Hypermetabolism in SIRS or MODS results in profound weight loss, cachexia, and loss of organ function. The goal of nutritional support is the preservation of organ structure and function. Although nutritional support may not definitely alter the course of organ dysfunction, it prevents generalized nutritional deficiencies and preserves gut integrity. Enteral nutrition may exert a physiologic effect that downregulates the systemic immune response and reduces oxidate stress.

The client diagnosed with ARDS is in respiratory distress and the ventilator is malfunctioning. Which intervention should the nurse implement first? a. Notify the respiratory therapist immediately. b. Ventilate with a manual resuscitation bag. c. Request stat arterial blood gases. d. Auscultate the clients one sounds.

ANS: B If the ventilator system malfunctions, the nurse must ventilate the client with a manual resuscitation (Ambu) bag until the problem is resolved. SYNTHESIS

Gastric lavage and administration of activated charcoal are ordered for an unconscious patient who has been admitted to the emergency department (ED) after ingesting 30 lorazepam (Ativan) tablets. Which action should the nurse plan to do first? a. Insert a large-bore orogastric tube. b. Assist with intubation of the patient. c. Prepare a 60-mL syringe with saline. d. Give first dose of activated charcoal.

ANS: B In an unresponsive patient, intubation is done before gastric lavage and activated charcoal administration to prevent aspiration. The other actions will be implemented after intubation. DIF: Cognitive Level: Apply (application) REF: 1689

The nurse is providing care for a client admitted to the burn unit. Select the most appropriate statement that identifies the nutritional needs of the client. a. The client needs 100 cal/kg throughout hospitalization. b. The hypermetabolic state after a burn injury contributes to poor healing. c. A cool environment decreases caloric demand. d. Maintaining a hypermetabolic rate decreases the clients risk of infection.

ANS: B It burn injury causes a hypermetabolic state resulting in protein and lipid cannibalism that affects word healing. Calories need to be 1.5 to 2 times the basal metabolic rate, with at least 1.5 to 2g/kg of body weight of protein daily. Environmental temperature within normal range that's the body function efficiently and about caloric expenditure to healing normal physiological processes. If the temperature is too warm or too cold, the body gives energy to warming or cooling, which takes away from energy used for tissue repair. High metabolic rates increase the risk of infection. APPLICATION (Made Easy)

Which medication may be administered with a bronchodilator because it can cause bronchospasms? a. Beta 2 Agonists b. Mucloytics c. Anticholinergic agents d. Xanthines

ANS: B Mucolytics may be administered with a bronchodilator because it can cause bronchospasms and inhibit ciliary function. Treatment is considered effective when bronchorrhea develops and coughing occurs. Beta 2 Agonists are used to relax bronchial smooth muscle and dilate airways to prevent bronchospasms. Anticholinergic agents are used to block the constriction of bronchial smooth muscle and reduce mucus production. Xanthines are used to dilate bronchial smooth muscle and reverse diaphragmatic muscle fatigue.

A client with a fat embolism continues to be hypoxic following therapy with positive end-expiratory pressure. What is the priority intervention to reduce oxygen demand? a. Give diuretics. b. Give neuromuscular blockers. c. Put the head of the bed flat. d. Use bronchodilators.

ANS: B Neuromuscular blockers cause skeletal muscle paralysis, reducing the amount of oxygen used by the restless skeletal muscles. This should improve oxygenation. Diuretics can be administered to reduce pulmonary congestion, and the head of the bed should be partially elevated to facilitate diaphragm movement. Bronchodilators maybe used, but they typically don't have enough of an effect to reduce the amount of hypoxia present. However, diuretics, had elevation, and bronchodilators would improve oxygen delivery, not reduce oxygen demand. APPLICATION (Made Easy)

The difference between primary and secondary multiple organ dysfunction syndrome (MODS) is that primary MODS is the result of a. widespread systemic inflammation that results in dysfunction of organs not involved in the initial insult. b. direct organ injury. c. disorganization of the immune system response. d. widespread disruption of the coagulation system.

ANS: B Organ dysfunction may be the direct consequence of an initial insult (primary MODS) or can manifest latently and involve organs not directly affected in the initial insult (secondary MODS). Patients can experience both primary and secondary MODS. Primary MODS results from a welldefined insult in which organ dysfunction occurs early and is directly attributed to the insult itself.

Nursing management of the patient receiving a neuromuscular blocking agent should include a. withholding all sedation and narcotics. b. protecting the patient from the environment. c. keeping the patient supine. d. speaking to the patient only when necessary.

ANS: B Patient safety is a major concern for the patient receiving a neuromuscular blocking agent because these patients are unable to protect themselves from the environment. Special precautions should be taken to protect the patient at all times.

Patient safety precautions when working with oxygen involve a. observation for signs of oxygen introduced hyperventilation. b. restriction of smoking. c. removal of all oxygen devices when eating to prevent aspiration. d. administration of oxygen at the nurse's discretion.

ANS: B Patient safety precautions when working with oxygen involve administration of oxygen and monitoring of its effectiveness. Activities include restricting smoking, administering supplemental oxygen as ordered, observing for signs of oxygen induced hypoventilation, monitoring the patient's ability to tolerate removal of oxygen while eating, and changing the oxygen delivery device from a mask to nasal prongs during meals as tolerated.

A patient was admitted to the critical care unit after a left pneumonectomy. The patient is receiving 40% oxygen via a simple facemask. The morning chest radiography study reveals right lower lobe pneumonia. After eating breakfast, the patient suddenly vomits and aspirates. The single most important measure to prevent the spread of infection between staff and patients is a. respiratory isolation. b. hand washing. c. use of PPE. d. antibiotics.

ANS: B Proper hand hygiene is the single most important measure available to prevent the spread of bacteria from person to person.

Shock syndrome can best be described as a a. physiologic state resulting in hypotension and tachycardia. b. generalized systemic response to inadequate tissue perfusion. c. degenerative condition leading to death. d. condition occurring with hypovolemia that results in irreversible hypotension.

ANS: B Shock is a complex pathophysiologic process that often results in multiple organ dysfunction syndrome and death. All types of shock eventually result in ineffective tissue perfusion and the development of acute circulatory failure.

Signs of hypovolemia in the trauma patient include a. distended neck veins. b. a decreased level of consciousness. c. bounding radial and pedal pulses. d. a widening pulse pressure.

ANS: B Signs of underperfusion include flattened neck veins, a decreased level of consciousness, weak and thready peripheral pulses, and a narrowed pulse pressure.

Which intervention will the nurse include in the plan of care for a patient who has cardiogenic shock? a. Check temperature every 2 hours. b. Monitor breath sounds frequently. c. Maintain patient in supine position. d. Assess skin for flushing and itching.

ANS: B Since pulmonary congestion and dyspnea are characteristics of cardiogenic shock, the nurse should assess the breath sounds frequently. The head of the bed is usually elevated to decrease dyspnea in patients with cardiogenic shock. Elevated temperature and flushing or itching of the skin are not typical of cardiogenic shock. DIF: Cognitive Level: Apply (application) REF: 1633

Laboratory values for DIC show abnormalities in a. liver function tests. b. tests for renal function. c. platelet counts. d. blood glucose levels.

ANS: C Low platelet counts and elevated Ddimer concentrations and fibrinogen degradation products are clinical indicators of DIC.

Which medication is not recommended in the treatment of shock related lactic acidosis? a. Glucose b. Sodium bicarbonate c. Vasoconstrictor d. Large quantity of crystalloids fluids

ANS: B Sodium bicarbonate is not recommended in the treatment of shockrelated lactic acidosis. Glucose control to a target level of 140 to 180 mg/dL is recommended for all critically ill patients. Vasoconstrictor agents are used to increase afterload by increasing the systemic vascular resistance and improving the patient's blood pressure level. Crystalloids are balanced electrolyte solutions that may be hypotonic, isotonic, or hypertonic. Examples of crystalloid solutions used in shock situations are normal saline and lactated Ringer solution.

Which medication should the nurse anticipate the healthcare provider ordering for the client diagnosed with ARDS? a. An aminoglycoside anabiotic. b. A synthetic surfactant. c. A potassium cation. d. A nonsteroidal anti-inflammatory drug.

ANS: B Surfactant therapy may be prescribed to reduce the surface tension of the alveoli. The surfactant helps maintain open alveoli, decreases the work of breathing, improves compliance, and helps prevent atelectasis. SYNTHESIS

Which of the following diagnostic criteria is indicative of ARDS? a. Radiologic evidence of bibasilar atelectasis b. PaO2/FiO2 ratio less than or equal to 200 mm Hg c. Pulmonary artery wedge pressure greater than 18 mm Hg d. Increased static and dynamic compliance

ANS: B The Berlin Definition of ARDS is as follows: timing—within 1 week of known clinical insult or new or worsening respiratory symptoms; chest imaging—bilateral opacities not fully explained by effusions, lobar or lung collapse, or nodules; origin of edema—respiratory failure not fully explained by cardiac failure or fluid overload; need objective assessment to exclude hydrostatic edema if no risk factor present; oxygenation—mild (200 mg Hg less than PaO2/ FiO2 less than or equal to 300 mm Hg with positive endrespiratory airway pressure (PEEP) or constant positive airway pressure greater than or equal to 5 cm H2O), moderate (100 mg Hg less than PaO2/FiO2 less than or equal to 200 mm Hg with PEEP greater than or equal to 5 cm H2O), or severe (PaO2/FiO2 less than or equal to 100 mm Hg with PEEP greater than or equal to 5 cm H2O). The mortality rate for ARDS is estimated to be 34% to 58%.

A patient who is unconscious after a fall from a ladder is transported to the emergency department by emergency medical personnel. During the primary survey of the patient, the nurse should a. obtain a complete set of vital signs. b. obtain a Glasgow Coma Scale score. c. ask about chronic medical conditions. d. attach a cardiac electrocardiogram monitor.

ANS: B The Glasgow Coma Scale is included when assessing for disability during the primary survey. The other information is part of the secondary survey. DIF: Cognitive Level: Apply (application) REF: 1676

A patient with cardiogenic shock has the following vital signs: BP 102/50, pulse 128, respirations 28. The pulmonary artery wedge pressure (PAWP) is increased and cardiac output is low. The nurse will anticipate an order for which medication? a. 5% human albumin b. Furosemide (Lasix) IV c. Epinephrine (Adrenalin) drip d. Hydrocortisone (Solu-Cortef)

ANS: B The PAWP indicates that the patient's preload is elevated, and furosemide is indicated to reduce the preload and improve cardiac output. Epinephrine would further increase heart rate and myocardial oxygen demand. 5% human albumin would also increase the PAWP. Hydrocortisone might be considered for septic or anaphylactic shock. DIF: Cognitive Level: Apply (application) REF: 1645

The nurse is aware that which diagnosis is most likely to contribute to the development of acute respiratory distress syndrome (ARDS)? a. Appendicitis b. Massive trauma c. Receiving conscious sedation d. Right meniscus injury

ANS: B The client with massive trauma will require multiple transfusions. Blood products are preserved with so I treat, which causes increased permeability in the lungs, the defect that allows ARDS to develop. Appendicitis, unless it causes overwhelming sepsis, won't lead to ARDS. Conscious sedation and injuries to the meniscus don't need to ARDS. ANALYSIS (Made Easy)

Which laboratory result would the nurse expect in the client diagnosed with DIC? a. a decreased prothrombin time. b. a low fibrinogen level. c. an increased platelet count. d. an increased white blood cell count.

ANS: B The fibrinogen level helps predict bleeding in DIC. As it becomes lower, the risk of bleeding increases. ANALYSIS

The major hemodynamic consequence of a massive pulmonary embolus is a. increased systemic vascular resistance leading to left heart failure. b. pulmonary hypertension leading to right heart failure. c. portal vein blockage leading to ascites. d. embolism to the internal carotids leading to a stroke.

ANS: B The major hemodynamic consequence of a pulmonary embolus is the development of pulmonary hypertension, which is part of the effect of a mechanical obstruction when more than 50% of the vascular bed is occluded. In addition, the mediators released at the injury site and the development of hypoxia cause pulmonary vasoconstriction, which further exacerbates pulmonary hypertension.

8. Which of the following statements is correct concerning endotracheal tube cuff management? a. The cuff should be deflated every hour to minimize pressure on the trachea. b. A small leak should be heard on inspiration if the cuff has been inflated using the minimal leak technique. c. Cuff pressures should be kept between 40 to 50 mm Hg to ensure an adequate seal. d. Cuff pressure monitoring should be done once every 24 hours.

ANS: B The minimal leak technique consists of injecting air into the cuff until no leak is heard and then withdrawing the air until a small leak is heard on inspiration. Problems with this technique include difficulty maintaining positive endexpiratory pressure and aspiration around the cuff.

The patients at highest risk for neurogenic shock are those who have had a. a stroke. b. a spinal cord injury. c. GuillainBarré syndrome. d. a craniotomy.

ANS: B The most common cause is spinal cord injury (SCI). Neurogenic shock may mistakenly be referred to as spinal shock. The latter condition refers to loss of neurologic activity below the level of SCI, but it does not necessarily involve ineffective tissue perfusion.

The most accurate and reliable control of FiO2 can be achieved through the use of a(n) a. simple mask. b. nonrebreathing circuit (closed). c. airentrainment mask. d. nonrebreathing mask.

ANS: B The most reliable and accurate means of delivering a prescribed concentration of oxygen is through the use of a nonrebreathing circuit (closed).

The nurse is caring for a patient who is intubated and receiving positive pressure ventilation to treat acute respiratory distress syndrome (ARDS). Which finding is most important to report to the health care provider? a. Blood urea nitrogen (BUN) level 32 mg/dL b. Red-brown drainage from orogastric tube c. Scattered coarse crackles heard throughout lungs d. Arterial blood gases: pH 7.31, PaCO2 50, PaO2 68

ANS: B The nasogastric drainage indicates possible gastrointestinal bleeding and/or stress ulcer, and should be reported. The pH and PaCO2 are slightly abnormal, but current guidelines advocating for permissive hypercapnia indicate that these would not indicate an immediate need for a change in therapy. The BUN is slightly elevated but does not indicate an immediate need for action. Adventitious breath sounds are commonly heard in patients with ARDS. DIF: Cognitive Level: Analyze (analysis) REF: 1668

The oxygen saturation (SpO2) for a patient with left lower lobe pneumonia is 90%. The patient has rhonchi, a weak cough effort, and complains of fatigue. Which action is a priority for the nurse to take? a. Position the patient on the left side. b. Assist the patient with staged coughing. c. Place a humidifier in the patient's room. d. Schedule a 2-hour rest period for the patient.

ANS: B The patient's assessment indicates that assisted coughing is needed to help remove secretions, which will improve oxygenation. A 2-hour rest period at this time may allow the oxygen saturation to drop further. Humidification will not be helpful unless the secretions can be mobilized. Positioning on the left side may cause a further decrease in oxygen saturation because perfusion will be directed more toward the more poorly ventilated lung. DIF: Cognitive Level: Apply (application) REF: 1661-1662

A patient is admitted to the burn unit with burns to the head, face, and hands. Initially, wheezes are heard, but an hour later, the lung sounds are decreased and no wheezes are audible. What is the best action for the nurse to take? a. Encourage the patient to cough and auscultate the lungs again. b. Notify the health care provider and prepare for endotracheal intubation. c. Document the results and continue to monitor the patient's respiratory rate. d. Reposition the patient in high-Fowler's position and reassess breath sounds.

ANS: B The patient's history and clinical manifestations suggest airway edema and the health care provider should be notified immediately, so that intubation can be done rapidly. Placing the patient in a more upright position or having the patient cough will not address the problem of airway edema. Continuing to monitor is inappropriate because immediate action should occur. DIF: Cognitive Level: Apply (application) REF: 459

A patient with respiratory failure has a respiratory rate of 6 breaths/minute and an oxygen saturation (SpO2) of 88%. The patient is increasingly lethargic. Which intervention will the nurse anticipate? a. Administration of 100% oxygen by non-rebreather mask b. Endotracheal intubation and positive pressure ventilation c. Insertion of a mini-tracheostomy with frequent suctioning d. Initiation of continuous positive pressure ventilation (CPAP)

ANS: B The patient's lethargy, low respiratory rate, and SpO2 indicate the need for mechanical ventilation with ventilator controlled respiratory rate. Administration of high flow oxygen will not be helpful because the patient's respiratory rate is so low. Insertion of a mini-tracheostomy will facilitate removal of secretions, but it will not improve the patient's respiratory rate or oxygenation. CPAP requires that the patient initiate an adequate respiratory rate to allow adequate gas exchange. DIF: Cognitive Level: Apply (application) REF: 1658 | 1662

A patient with septicemia develops prolonged bleeding from venipuncture sites and blood in the stools. Which action is most important for the nurse to take? a. Avoid venipunctures. b. Notify the patient's physician. c. Apply sterile dressings to the sites. d. Give prescribed proton-pump inhibitors.

ANS: B The patient's new onset of bleeding and diagnosis of sepsis suggest that disseminated intravascular coagulation (DIC) may have developed, which will require collaborative actions such as diagnostic testing, blood product administration, and heparin administration. The other actions also are appropriate, but the most important action should be to notify the physician so that DIC treatment can be initiated rapidly. DIF: Cognitive Level: Apply (application) REF: 658-659

A patient is admitted to the emergency department (ED) after falling through the ice while ice skating. Which assessment will the nurse obtain first? a. Heart rate b. Breath sounds c. Body temperature d. Level of consciousness

ANS: B The priority assessment relates to ABCs (airway, breathing, circulation) and how well the patient is oxygenating, so breath sounds should be assessed first. The other data will also be collected rapidly but are not as essential as the breath sounds. DIF: Cognitive Level: Apply (application) REF: 1685

A patient was admitted to the critical care unit with acute respiratory failure. The patient has been on a ventilator for 3 days and is being considered for weaning. The ventilator highpressure alarm keeps going off. When you enter the room, the ventilator inoperative alarm sounds. Which of the following criteria would indicate that the patient is ready to be weaned? a. FiO2 greater than 50% b. Rapid shallow breathing index less than 105 c. Minute ventilation greater than 10 L/min d. Vital capacity/kg greater than or equal to 15 mL

ANS: B The rapid shallow breathing index (RSBI) has been shown to be predictive of weaning success. To calculate the RSBI, the patient's respiratory rate and minute ventilation are measured for 1 minute during spontaneous breathing. The measured respiratory rate is then divided by the tidal volume (expressed in liters). An RSBI less than 105 is considered predictive of weaning success. If the patient meets criteria for weaning readiness and has an RSBI less than 105, a spontaneous breathing trial can be performed.

A nurse is caring for a patient who has burns of the ears, head, neck, and right arm and hand. The nurse should place the patient in which position? a. Place the right arm and hand flexed in a position of comfort. b. Elevate the right arm and hand on pillows and extend the fingers. c. Assist the patient to a supine position with a small pillow under the head. d. Position the patient in a side-lying position with rolled towel under the neck.

ANS: B The right hand and arm should be elevated to reduce swelling and the fingers extended to avoid flexion contractures (even though this position may not be comfortable for the patient). The patient with burns of the ears should not use a pillow for the head because this will put pressure on the ears, and the pillow may stick to the ears. Patients with neck burns should not use a pillow because the head should be maintained in an extended position in order to avoid contractures. DIF: Cognitive Level: Apply (application) REF: 462

A nurse is caring for a patient with ARDS who is being treated with mechanical ventilation and high levels of positive end-expiratory pressure (PEEP). Which assessment finding by the nurse indicates that the PEEP may need to be reduced? a. The patient's PaO2 is 50 mm Hg and the SaO2 is 88%. b. The patient has subcutaneous emphysema on the upper thorax. c. The patient has bronchial breath sounds in both the lung fields. d. The patient has a first-degree atrioventricular heart block with a rate of 58.

ANS: B The subcutaneous emphysema indicates barotrauma caused by positive pressure ventilation and PEEP. Bradycardia, hypoxemia, and bronchial breath sounds are all concerns and will need to be addressed, but they are not specific indications that PEEP should be reduced. DIF: Cognitive Level: Apply (application) REF: 1668-1669

The therapeutic blood level for theophylline (Xanthines) is a. 5 to 10 mg/dL. b. 10 to 20 mg/dL. c. 20 to 30 mg/dL. d. 35 to 45 mg/dL.

ANS: B Therapeutic blood level for theophylline is 10 to 20 mg/dL.

A young adult patient who is in the rehabilitation phase 6 months after a severe face and neck burn tells the nurse, "I'm sorry that I'm still alive. My life will never be normal again." Which response by the nurse is best? a. "Most people recover after a burn and feel satisfied with their lives." b. "It's true that your life may be different. What concerns you the most?" c. "It is really too early to know how much your life will be changed by the burn." d. "Why do you feel that way? You will be able to adapt as your recovery progresses."

ANS: B This response acknowledges the patient's feelings and asks for more assessment data that will help in developing an appropriate plan of care to assist the patient with the emotional response to the burn injury. The other statements are accurate, but do not acknowledge the anxiety and depression that the patient is expressing. DIF: Cognitive Level: Apply (application) REF: 470-471

While the patient's full-thickness burn wounds to the face are exposed, what is the best nursing action to prevent cross contamination? a. Use sterile gloves when removing old dressings. b. Wear gowns, caps, masks, and gloves during all care of the patient. c. Administer IV antibiotics to prevent bacterial colonization of wounds. d. Turn the room temperature up to at least 70° F (20° C) during dressing changes.

ANS: B Use of gowns, caps, masks, and gloves during all patient care will decrease the possibility of wound contamination for a patient whose burns are not covered. When removing contaminated dressings and washing the dirty wound, use nonsterile, disposable gloves. The room temperature should be kept at approximately 85° F for patients with open burn wounds to prevent shivering. Systemic antibiotics are not well absorbed into deep burns because of the lack of circulation. DIF: Cognitive Level: Apply (application) REF: 461

After receiving 2 L of normal saline, the central venous pressure for a patient who has septic shock is 10 mm Hg, but the blood pressure is still 82/40 mm Hg. The nurse will anticipate an order for a. nitroglycerine (Tridil). b. norepinephrine (Levophed). c. sodium nitroprusside (Nipride). d. methylprednisolone (Solu-Medrol).

ANS: B When fluid resuscitation is unsuccessful, vasopressor drugs are administered to increase the systemic vascular resistance (SVR) and blood pressure, and improve tissue perfusion. Nitroglycerin would decrease the preload and further drop cardiac output and BP. Methylprednisolone (Solu-Medrol) is considered if blood pressure does not respond first to fluids and vasopressors. Nitroprusside is an arterial vasodilator and would further decrease SVR. DIF: Cognitive Level: Apply (application) REF: 1643

When assessing a patient who spilled hot oil on the right leg and foot, the nurse notes that the skin is dry, pale, hard skin. The patient states that the burn is not painful. What term would the nurse use to document the burn depth? a. First-degree skin destruction b. Full-thickness skin destruction c. Deep partial-thickness skin destruction d. Superficial partial-thickness skin destruction

ANS: B With full-thickness skin destruction, the appearance is pale and dry or leathery and the area is painless because of the associated nerve destruction. Erythema, swelling, and blisters point to a deep partial-thickness burn. With superficial partial-thickness burns, the area is red, but no blisters are present. First-degree burns exhibit erythema, blanching, and pain. DIF: Cognitive Level: Understand (comprehension) REF: 454

After change-of-shift report in the progressive care unit, who should the nurse care for first? a. Patient who had an inferior myocardial infarction 2 days ago and has crackles in the lung bases b. Patient with suspected urosepsis who has new orders for urine and blood cultures and antibiotics c. Patient who had a T5 spinal cord injury 1 week ago and currently has a heart rate of 54 beats/minute d. Patient admitted with anaphylaxis 3 hours ago who now has clear lung sounds and a blood pressure of 108/58 mm Hg

ANS: B Antibiotics should be administered within the first hour for patients who have sepsis or suspected sepsis in order to prevent progression to systemic inflammatory response syndrome (SIRS) and septic shock. The data on the other patients indicate that they are more stable. Crackles heard only at the lung bases do not require immediate intervention in a patient who has had a myocardial infarction. Mild bradycardia does not usually require atropine in patients who have a spinal cord injury. The findings for the patient admitted with anaphylaxis indicate resolution of bronchospasm and hypotension. DIF: Cognitive Level: Analyze (analysis) REF: 1644 | 1646

The nurse is planning care for a client with the late-stage burn wound to promote healing. What is the most important intervention for the nurse to include? a. Removing eschar tissue from the skin b. I'm playing continuous compression wraps c. Wearing clothing to protect the burn from the sun d. Maintaining wound care irrigation

ANS: B Applying continuous compression wraps aid skin healing and prevent hypertrophied tissue from forming. The other interventions are appropriate for the client with the burn wound but doesn't necessarily help minimize scarring. APPLICATION (Made Easy)

The client has sustained a burn wound. What is the most important intervention by the nurse to decrease hypertrophied scarring during later stages of healing? a. Remove all tissue in the wound area. b. Apply continuous pressure using elastic wraps. c. Where clothing to protect the burn from the sun. d. Maintain wound dressing changes.

ANS: B Using elastic wraps and bandages to apply continuous pressure during the early stages of wound healing can help prevent keloid scar formation. Removing tissue, especially eschar, promotes wound healing as do dressing changes, but neither directly decrease the scar formation. Wearing clothing prevent sunburn but doesn't decrease scar formation. APPLICATION (Made Easy)

Which actions should the nurse initiate to reduce the risk for ventilator-associated pneumonia (VAP) (select all that apply)? a. Obtain arterial blood gases daily. b. Provide a "sedation holiday" daily. c. Elevate the head of the bed to at least 30°. d. Give prescribed pantoprazole (Protonix). e. Provide oral care with chlorhexidine (0.12%) solution daily.

ANS: B, C, D, E All of these interventions are part of the ventilator bundle that is recommended to prevent VAP. Arterial blood gases may be done daily but are not always necessary and do not help prevent VAP. DIF: Cognitive Level: Apply (application) REF: 1668

Which of the following findings confirms the diagnosis of a PE? a. Lowprobability V/Q scan b. Negative pulmonary angiogram c. Highprobability V/Q scan d. Absence of vascular markings on the chest radiograph

ANS: C A definitive diagnosis of a pulmonary embolus requires confirmation by a highprobability V/Q scan, an abnormal pulmonary angiogram or computed tomography scan, or strong clinical suspicion coupled with abnormal findings on lower extremity deep venous thrombosis studies.

A patient with burns covering 40% total body surface area (TBSA) is in the acute phase of burn treatment. Which snack would be best for the nurse to offer to this patient? a. Bananas b. Orange gelatin c. Vanilla milkshake d. Whole grain bagel

ANS: C A patient with a burn injury needs high protein and calorie food intake, and the milkshake is the highest in these nutrients. The other choices are not as nutrient-dense as the milkshake. Gelatin is likely high in sugar. The bagel is a good carbohydrate choice, but low in protein. Bananas are a good source of potassium, but are not high in protein and calories. DIF: Cognitive Level: Apply (application) REF: 467

The emergency department (ED) nurse receives report that a patient involved in a motor vehicle crash is being transported to the facility with an estimated arrival in 1 minute. In preparation for the patient's arrival, the nurse will obtain a. hypothermia blanket. b. lactated Ringer's solution. c. two 14-gauge IV catheters. d. dopamine (Intropin) infusion.

ANS: C A patient with multiple trauma may require fluid resuscitation to prevent or treat hypovolemic shock, so the nurse will anticipate the need for 2 large bore IV lines to administer normal saline. Lactated Ringer's solution should be used cautiously and will not be ordered until the patient has been assessed for possible liver abnormalities. Vasopressor infusion is not used as the initial therapy for hypovolemic shock. Patients in shock need to be kept warm not cool. DIF: Cognitive Level: Apply (application) REF: 1641

Indications to support a pneumonectomy are a. lesions confined to a single lobe. b. bronchiectasis. c. unilateral tuberculosis. d. lung abscesses or cyst.

ANS: C A pneumonectomy is the removal of entire lung with or without resection of the mediastinal lymph nodes. Indications include malignant lesions, unilateral tuberculosis, extensive unilateral bronchiectasis, multiple lung abscesses, massive hemoptysis, and bronchopleural fistula.

A pneumothorax greater than 15% requires a. systemic antibiotics to treat the inflammatory response. b. an occlusive dressing to equalize lung pressures. c. interventions to evacuate the air from the pleural space and facilitate reexpansion of the collapsed lung. d. mechanical ventilation to assist with reexpansion of the collapsed lung.

ANS: C A pneumothorax greater than 15% requires intervention to evacuate the air from the pleural space and facilitate reexpansion of the collapsed lung. Interventions include aspiration of the air with a needle and placement of a smallbore (12-20 Fr) or largebore (24-40 Fr) chest tube.

A patient was admitted to the critical care unit after a left pneumonectomy. The patient is receiving 40% oxygen via a simple facemask. The morning chest radiography study reveals right lower lobe pneumonia. After eating breakfast, the patient suddenly vomits and aspirates. Which test would the nurse expect the health care provider to order to identify the infectious pathogen? a. CBC with differential b. Wound culture of surgical site c. Sputum Gram stain and culture d. Urine specimen

ANS: C A sputum Gram stain and culture are done to facilitate the identification of the infectious pathogen. In 50% of cases, though, a causative agent is not identified. A diagnostic bronchoscopy may be needed, particularly if the diagnosis is unclear or current therapy is not working. In addition, a complete blood count with differential, chemistry panel, blood cultures, and arterial blood gas analysis is obtained.

A pregnant client arrives at the emergency department with abruptio placentae at 34 weeks' gestation. The nurse is aware that the client is at risk for developing: a. thrombocytopenia b. ITP c. DIC d. HATT

ANS: C Abruptio placentae is a cause of DIC because of activation of the clotting cascade after hemorrhage. APPLICATION (Made Easy)

A client received burns to his entire back and left arm. The nurse uses the rule of nines to calculate that he has sustained burns to what percentage of his body? a. 9% b. 18% c. 27% d. 36%

ANS: C According to the rule of nines, posterior trunk, anterior trunk, and legs are each 18% of the total body surface. The head, neck, and arms are each 9% of total body surface, and the perineum is 1%. In this case, the client received it burns to his back 18% and one arm 9%, totaling 27% of his body. APPLICATION (Made Easy)

A patient with extensive electrical burn injuries is admitted to the emergency department. Which prescribed intervention should the nurse implement first? a. Assess oral temperature. b. Check a potassium level. c. Place on cardiac monitor. d. Assess for pain at contact points.

ANS: C After an electrical burn, the patient is at risk for fatal dysrhythmias and should be placed on a cardiac monitor. Assessing the oral temperature is not as important as assessing for cardiac dysrhythmias. Checking the potassium level is important. However, it will take time before the laboratory results are back. The first intervention is to place the patient on a cardiac monitor and assess for dysrhythmias, so that they can be treated if occurring. A decreased or increased potassium level will alert the nurse to the possibility of dysrhythmias. The cardiac monitor will alert the nurse immediately of any dysrhythmias. Assessing for pain is important, but the patient can endure pain until the cardiac monitor is attached. Cardiac dysrhythmias can be lethal. DIF: Cognitive Level: Analyze (analysis) REF: 452 | 456

During change-of-shift report on a medical unit, the nurse learns that a patient with aspiration pneumonia who was admitted with respiratory distress has become increasingly agitated. Which action should the nurse take first? a. Give the prescribed PRN sedative drug. b. Offer reassurance and reorient the patient. c. Use pulse oximetry to check the oxygen saturation. d. Notify the health care provider about the patient's status.

ANS: C Agitation may be an early indicator of hypoxemia. The other actions may also be appropriate, depending on the findings about oxygen saturation. DIF: Cognitive Level: Apply (application) REF: 1663

A patient has just arrived in the emergency department after an electrical burn from exposure to a high-voltage current. What is the priority nursing assessment? a. Oral temperature b. Peripheral pulses c. Extremity movement d. Pupil reaction to light

ANS: C All patients with electrical burns should be considered at risk for cervical spine injury, and assessments of extremity movement will provide baseline data. The other assessment data are also necessary but not as essential as determining the cervical spine status. DIF: Cognitive Level: Apply (application) REF: 452

A client found unconscious at home is brought to the emergency department. Physical examination shows cherry-red mucous membranes, nailbeds, and skin. The nurse interprets these findings as indicating which of the following? a. Spider bite b. Aspirin ingestion c. Hydrocarbon ingestion d. Carbon monoxide poisoning

ANS: D Cherry-red skin indicates exposure to high levels of carbon monoxide. ANALYSIS (Made Easy)

Family members are in the patient's room when the patient has a cardiac arrest and the staff start resuscitation measures. Which action should the nurse takenext? a. Keep the family in the room and assign a staff member to explain the care given and answer questions. b. Ask the family to wait outside the patient's room with a designated staff member to provide emotional support. c. Ask the family members about whether they would prefer to remain in the patient's room or wait outside the room. d. Tell the family members that patients are comforted by having family members present during resuscitation efforts.

ANS: C Although many family members and patients report benefits from family presence during resuscitation efforts, the nurse's initial action should be to determine the preference of these family members. The other actions may be appropriate, but this will depend on what is learned when assessing family preferences. DIF: Cognitive Level: Apply (application) REF: 1679

Longterm ventilator management over 21 days is best handled through use of a(n) a. oropharyngeal airway. b. esophageal obturator airway. c. tracheostomy tube. d. endotracheal intubation.

ANS: C Although no ideal time to perform the procedure has been identified, it is commonly accepted that if a patient has been intubated or is anticipated to be intubated for longer than 7 to 10 days, a tracheostomy should be performed.

To evaluate the effectiveness of ordered interventions for a patient with ventilatory failure, which diagnostic test will be most useful to the nurse? a. Chest x-ray b. Oxygen saturation c. Arterial blood gas analysis d. Central venous pressure monitoring

ANS: C Arterial blood gas (ABG) analysis is most useful in this setting because ventilatory failure causes problems with CO2 retention, and ABGs provide information about the PaCO2 and pH. The other tests may also be done to help in assessing oxygenation or determining the cause of the patient's ventilatory failure. DIF: Cognitive Level: Apply (application) REF: 1661

While assessing a client with disseminated intravascular coagulation (DIC), the nurse suspects the client has developed internal bleeding. The nurse should assess the client for: a. hypertension b. petechiae c. increasing abdominal girth d. bradycardia

ANS: C As blood collects in the peritoneal cavity, dilation and distention of the abdomen occur/ This is reflected by an increase in abdominal girth. The client with DIC would have hypotension and tachycardia. ANALYSIS (Made Easy)

The client diagnosed with ARDS is transferred to the intensive care department and placed on a ventilator. Which intervention should the nurse implement first? a. Confirmed that the ventilator settings are correct. b. Verify that the ventilator alarms are functioning properly. c. Assess the respiratory status and pulse oximeter reading. d. Monitor the clients arterial blood gas results.

ANS: C Assessment is the first part of the nursing process and is the first intervention the nurse should implement and caring for a client on a ventilator. SYNTHESIS

Which assessment data indicate to the nurse the client diagnosed with ARDS has experienced a complication secondary to the ventilator? a. The clients urine output is 100 mL in four hours. b. The post oximeter reading is greater than 95%. c. The client has asymmetrical chest expansion. d. The telemetry reading shows sinus tachycardia.

ANS: C Asymmetrical chest expansion indicates the client has had a pneumothorax, which is a complication of mechanical ventilation. ANALYSIS

A 28-year-old patient who has deep human bite wounds on the left hand is being treated in the urgent care center. Which action will the nurse plan to take? a. Prepare to administer rabies immune globulin (BayRab). b. Assist the health care provider with suturing of the bite wounds. c. Teach the patient the reason for the use of prophylactic antibiotics. d. Keep the wounds dry until the health care provider can assess them.

ANS: C Because human bites of the hand frequently become infected, prophylactic antibiotics are usually prescribed to prevent infection. To minimize infection, deep bite wounds on the extremities are left open. Rabies immune globulin might be used after an animal bite. Initial treatment of bite wounds includes copious irrigation to help clean out contaminants and microorganisms. DIF: Cognitive Level: Apply (application) REF: 1688

A 22-year-old patient who experienced a near drowning accident in a local pool, but now is awake and breathing spontaneously, is admitted for observation. Which assessment will be most important for the nurse to take during the observation period? a. Auscultate heart sounds. b. Palpate peripheral pulses. c. Auscultate breath sounds. d. Check pupil reaction to light.

ANS: C Because pulmonary edema is a common complication after near drowning, the nurse should assess the breath sounds frequently. The other information also will be obtained by the nurse, but it is not as pertinent to the patient's admission diagnosis. DIF: Cognitive Level: Apply (application) REF: 1686

Which finding about a patient who is receiving vasopressin (Pitressin) to treat septic shock is most important for the nurse to communicate to the health care provider? a. The patient's urine output is 18 mL/hr. b. The patient's heart rate is 110 beats/minute. c. The patient is complaining of chest pain. d. The patient's peripheral pulses are weak.

ANS: C Because vasopressin is a potent vasoconstrictor, it may decrease coronary artery perfusion. The other information is consistent with the patient's diagnosis and should be reported to the health care provider but does not indicate a need for a change in therapy. DIF: Cognitive Level: Apply (application) REF: 1643

A patient with hypotension and an elevated temperature after working outside on a hot day is treated in the emergency department (ED). The nurse determines that discharge teaching has been effective when the patient makes which statement? a. "I will take salt tablets when I work outdoors in the summer." b. "I should take acetaminophen (Tylenol) if I start to feel too warm." c. "I should drink sports drinks when working outside in hot weather." d. "I will move to a cool environment if I notice that I am feeling confused."

ANS: C Electrolyte solutions such as sports drinks help replace fluid and electrolytes lost when exercising in hot weather. Salt tablets are not recommended because of the risks of gastric irritation and hypernatremia. Antipyretic medications are not effective in lowering body temperature elevations caused by excessive exposure to heat. A patient who is confused is likely to have more severe hyperthermia and will be unable to remember to take appropriate action. DIF: Cognitive Level: Apply (application) REF: 1682

Which collaborative treatment would the nurse anticipate for the client diagnosed with DIC? a. administer oral anticoagulants b. prepare for plasmapheresis c. administer frozen plasma d. calculate the intake and output

ANS: C Fresh frozen plasma and platelet concentrates are administered to restore clotting factors and platelets. SYNTHESIS

A patient with severe burns has crystalloid fluid replacement ordered using the Parkland formula. The initial volume of fluid to be administered in the first 24 hours is 30,000 mL. The initial rate of administration is 1875 mL/hr. After the first 8 hours, what rate should the nurse infuse the IV fluids? a. 350 mL/hour b. 523 mL/hour c. 938 mL/hour d. 1250 mL/hour

ANS: C Half of the fluid replacement using the Parkland formula is administered in the first 8 hours and the other half over the next 16 hours. In this case, the patient should receive half of the initial rate, or 938 mL/hr. DIF: Cognitive Level: Apply (application) REF: 460

The nurse is reviewing laboratory results on a patient who had a large burn 48 hours ago. Which result requires priority action by the nurse? a. Hematocrit 53% b. Serum sodium 147 mEq/L c. Serum potassium 6.1 mEq/L d. Blood urea nitrogen 37 mg/dL

ANS: C Hyperkalemia can lead to fatal dysrhythmias and indicates that the patient requires cardiac monitoring and immediate treatment to lower the potassium level. The other laboratory values are also abnormal and require changes in treatment, but they are not as immediately life threatening as the elevated potassium level. DIF: Cognitive Level: Apply (application) REF: 458

Hypovolemic shock that results from an internal shifting of fluid from the intravascular space to the extravascular space is known as a. absolute hypovolemia. b. distributive hypovolemia. c. relative hypovolemia. d. compensatory hypovolemia.

ANS: C Hypovolemia results in a loss of circulating fluid volume. A decrease in circulating volume leads to a decrease in venous return, which results in a decrease in end diastolic volume or preload.

Nursing interventions to limit the complications of suctioning include a. inserting the suction catheter no more than 5 inches. b. premedicating the patient with atropine. c. hyperoxygenating the patient with 100% oxygen. d. increasing the suction to 150 mm Hg.

ANS: C Hypoxemia can be minimized by giving the patient three hyperoxygenation breaths (breaths at 100% FiO2) with the ventilator before the procedure and again after each pass of the suction catheter.

Which of the following causes of hypoxemia is the result of blood passing through unventilated portions of the lungs? a. Alveolar hypoventilation b. Dead space ventilation c. Intrapulmonary shunting d. Drug overdose

ANS: C Hypoxemia is the result of impaired gas exchange and is the hallmark of acute respiratory failure. Hypercapnia may be present, depending on the underlying cause of the problem. The main causes of hypoxemia are alveolar hypoventilation, ventilation/perfusion (V/Q) mismatching, and intrapulmonary shunting. Intrapulmonary shunting occurs when blood passes through a portion of a lung that is not ventilated. Drug overdose is an extrapulmonary cause that affects the brain.

According to the National Association of Medical Direction of Respiratory Care Consensus Panel prolonged mechanical ventilation has been defined as a. "the need for > or = to 30 consecutive days of mechanical ventilation for > or = to 8 hours per day" b. "the need for > or = to 15 consecutive days of mechanical ventilation for > or = to 12 hours per day" c. "the need for > or = to 21 consecutive days of mechanical ventilation for > or = to 6 hours per day" d. "the need for > or = to 40 consecutive days of mechanical ventilation for > or = to 4 hours per day"

ANS: C In 2005, the National Association for Medical Direction of Respiratory Care Consensus Panel recommended that LTMVD (which they referred to as prolonged mechanical ventilation) be defined as "the need for > or = to 21 consecutive days of mechanical ventilation for > or = to 6 hours per day."

The nurse is explaning the process of acute respiratory distress syndrome (ARDS) to a client. What is the best explanation for the nurse to tell the client? a. Alveoli are overexpanded b. Alveoli increase perfusion c. Alveolar spaces are filled with fluid d. Alveoli improve gaseous exchange

ANS: C In ARDS, the alveolar membranes are more permeable and the spaces are fluid filled. APPLICATION (Made Easy)

A client has just been admitted to the hospital after sustaining partial-thickness burns to both lower extremities and portions of the trunk. The nurse is aware that the most important IV fluid it to administer is: a. Albumin. b. Dextrose 5% in water. c. lactated ringer's solution. d. Normal saline solution with 2 mEq of potassium per 100 mL.

ANS: C Lactated ringer's solution replaces lost sodium and corrects metabolic acidosis, both of which commonly occur following a burn. I'll be human is used as adjunct therapy, not primary fluid replacement. Dextrose isn't given to burn clients during the first 24 hours because it can cause pseudodiabetes. The client is hyperkalemic from the potassium shift from the intracellular space to the plasma, so potassium would be detrimental. APPLICATION (Made Easy)

A 19-year-old patient with massive trauma and possible spinal cord injury is admitted to the emergency department (ED). Which assessment finding by the nurse will help confirm a diagnosis of neurogenic shock? a. Inspiratory crackles. b. Cool, clammy extremities. c. Apical heart rate 45 beats/min. d. Temperature 101.2° F (38.4° C).

ANS: C Neurogenic shock is characterized by hypotension and bradycardia. The other findings would be more consistent with other types of shock. DIF: Cognitive Level: Understand (comprehension) REF: 1634

An older patient with cardiogenic shock is cool and clammy and hemodynamic monitoring indicates a high systemic vascular resistance (SVR). Which intervention should the nurse anticipate doing next? a. Increase the rate for the dopamine (Intropin) infusion. b. Decrease the rate for the nitroglycerin (Tridil) infusion. c. Increase the rate for the sodium nitroprusside (Nipride) infusion. d. Decrease the rate for the 5% dextrose in normal saline (D5/0.9% NS) infusion.

ANS: C Nitroprusside is an arterial vasodilator and will decrease the SVR and afterload, which will improve cardiac output. Changes in the D5/0.9% NS and nitroglycerin infusions will not directly decrease SVR. Increasing the dopamine will tend to increase SVR. DIF: Cognitive Level: Apply (application) REF: 1644

The majority of falls accounting for traumatic injury occur in what population? a. Construction workers b. Adolescents c. Older adults d. Young adults

ANS: C Older persons experience most of the falls that result in injuries, and these falls are likely to occur from level surfaces or steps. Because many of the falls may be caused by an underlying medical condition (e.g., syncope, myocardial infarction, dysrhythmias), management of an older patient who has fallen must include an evaluation of events and conditions immediately preceding the fall.

One theory suggests that organ dysfunction in MODS occurs in a sequential or progressive pattern. Place the following organs in the order in which they are affected: 1. Bone marrow 2. Cardiac 3. Gut 4. Kidneys 5. Liver 6. Lungs a. 6, 5, 2, 1, 3, 4 b. 5, 4, 6, 1, 2, 3 c. 6, 5, 3, 4, 2, 1 d. 6, 3, 4, 5, 2, 1

ANS: C Organ dysfunction may occur in a sequential or progressive pattern. Organ dysfunction may begin in the lungs, the most commonly affected major organ, and progress to the liver, gut, and kidneys. Cardiac and bone marrow dysfunction may follow. Neurologic and autonomic system impairment may occur and propagate the progression of organ failure, which is associated with illness severity and mortality. Organs may fail simultaneously; for example, kidney dysfunction may occur concurrently with hepatic dysfunction. After the initial insult and resuscitation, patients develop persistent hypermetabolism, a metabolic consequence of sustained systemic inflammation and physiologic stress followed closely by pulmonary dysfunction, manifested as acute respiratory distress syndrome.

The two most common causes of hospitalacquired pneumonia in the United States are a. Staphylococcus aureus and Pseudomonas aeruginosa b. Escherichia coli and Haemophilus influenzae c. methicillinresistant Staphylococcus aureus and Pseudomonas aeruginosa d. Klebsiella spp. and Enterobacter spp.

ANS: C Pathogens that can cause health care-associated pneumonia are similar to those causing both community and hospitalacquired pneumonia (HAP) with Pseudomonas aeruginosa and methicillinresistant Staphylococcus aureus (MRSA) being the most common in the United States. Pathogens that can cause HAP include Escherichia coli, Haemophilus influenzae, methicillinsensitive Staphylococcus aureus, Streptococcus pneumoniae,P. aeruginosa, Acinetobacter baumannii, MRSA, Klebsiella spp., andEnterobacter spp.

A patient develops increasing dyspnea and hypoxemia 2 days after heart surgery. To determine whether the patient has acute respiratory distress syndrome (ARDS) or pulmonary edema caused by heart failure, the nurse will plan to assist with a. obtaining a ventilation-perfusion scan. b. drawing blood for arterial blood gases. c. insertion of a pulmonary artery catheter. d. positioning the patient for a chest x-ray.

ANS: C Pulmonary artery wedge pressures are normal in the patient with ARDS because the fluid in the alveoli is caused by increased permeability of the alveolar-capillary membrane rather than by the backup of fluid from the lungs (as occurs in cardiogenic pulmonary edema). The other tests will not help in differentiating cardiogenic from noncardiogenic pulmonary edema. DIF: Cognitive Level: Apply (application) REF: 1667

The PassyMuir valve is contraindicated in patients a. who are trying to relearn normal breathing patterns. b. who experience low secretions. c. with laryngeal or pharyngeal dysfunction. d. who want to speak while on the ventilator.

ANS: C The PassyMuir valve is contraindicated in patients with laryngeal or pharyngeal dysfunction, excessive secretions, or poor lung compliance.

The patient with neurogenic shock is receiving a phenylephrine (Neo-Synephrine) infusion through a right forearm IV. Which assessment finding obtained by the nurse indicates a need for immediate action? a. The patient's heart rate is 58 beats/minute. b. The patient's extremities are warm and dry. c. The patient's IV infusion site is cool and pale. d. The patient's urine output is 28 mL over the last hour.

ANS: C The coldness and pallor at the infusion site suggest extravasation of the phenylephrine. The nurse should discontinue the IV and, if possible, infuse the medication into a central line. An apical pulse of 58 is typical for neurogenic shock but does not indicate an immediate need for nursing intervention. A 28-mL urinary output over 1 hour would require the nurse to monitor the output over the next hour, but an immediate change in therapy is not indicated. Warm, dry skin is consistent with early neurogenic shock, but it does not indicate a need for a change in therapy or immediate action. DIF: Cognitive Level: Apply (application) REF: 1643

A patient who has been involved in a motor vehicle crash arrives in the emergency department (ED) with cool, clammy skin; tachycardia; and hypotension. Which intervention ordered by the health care provider should the nurse implement first? a. Insert two large-bore IV catheters. b. Initiate continuous electrocardiogram (ECG) monitoring. c. Provide oxygen at 100% per non-rebreather mask. d. Draw blood to type and crossmatch for transfusions.

ANS: C The first priority in the initial management of shock is maintenance of the airway and ventilation. ECG monitoring, insertion of IV catheters, and obtaining blood for transfusions should also be rapidly accomplished but only after actions to maximize oxygen delivery have been implemented. DIF: Cognitive Level: Apply (application) REF: 1641

After reviewing the information shown in the accompanying figure for a patient with pneumonia and sepsis, which information is most important to report to the health care provider? a. Temperature and IV site appearance b. Oxygen saturation and breath sounds c. Platelet count and presence of petechiae d. Blood pressure, pulse rate, respiratory rate.

ANS: C The low platelet count and presence of petechiae suggest that the patient may have disseminated intravascular coagulation and that multiple organ dysfunction syndrome (MODS) is developing. The other information will also be discussed with the health care provider but does not indicate that the patient's condition is deteriorating or that a change in therapy is needed immediately. DIF: Cognitive Level: Analyze (analysis) REF: 1640

Depending on the patient's risk for the recurrence of PE, a patient may be placed on warfarin for a. 1 to 3 months. b. 3 to 6 months. c. 3 to 12 months. d. 12 to 36 months.

ANS: C The patient should remain on warfarin for 3 to 12 months depending on his or her risk for thromboembolic disease.

A patient with possible disseminated intravascular coagulation arrives in the emergency department with a blood pressure of 82/40, temperature 102° F (38.9° C), and severe back pain. Which physician order will the nurse implement first? a. Administer morphine sulfate 4 mg IV. b. Give acetaminophen (Tylenol) 650 mg. c. Infuse normal saline 500 mL over 30 minutes. d. Schedule complete blood count and coagulation studies.

ANS: C The patient's blood pressure indicates hypovolemia caused by blood loss and should be addressed immediately to improve perfusion to vital organs. The other actions also are appropriate and should be rapidly implemented, but improving perfusion is the priority for this patient. DIF: Cognitive Level: Apply (application) REF: 659

The nurse documents the vital signs for a patient admitted 2 days ago with gram-negative sepsis: temperature 101.2° F, blood pressure 90/56 mm Hg, pulse 92, respirations 34. Which action should the nurse take next? a. Give the scheduled IV antibiotic. b. Give the PRN acetaminophen (Tylenol). c. Obtain oxygen saturation using pulse oximetry. d. Notify the health care provider of the patient's vital signs.

ANS: C The patient's increased respiratory rate in combination with the admission diagnosis of gram-negative sepsis indicates that acute respiratory distress syndrome (ARDS) may be developing. The nurse should check for hypoxemia, a hallmark of ARDS. The health care provider should be notified after further assessment of the patient. Giving the scheduled antibiotic and the PRN acetaminophen will also be done, but they are not the highest priority for a patient who may be developing ARDS. DIF: Cognitive Level: Apply (application) REF: 1665 | 1667

On admission to the burn unit, a patient with an approximate 25% total body surface area (TBSA) burn has the following initial laboratory results: Hct 58%, Hgb 18.2 mg/dL (172 g/L), serum K+ 4.9 mEq/L (4.8 mmol/L), and serum Na+ 135 mEq/L (135 mmol/L). Which action will the nurse anticipate taking now? a. Monitor urine output every 4 hours. b. Continue to monitor the laboratory results. c. Increase the rate of the ordered IV solution. d. Type and crossmatch for a blood transfusion.

ANS: C The patient's laboratory data show hemoconcentration, which may lead to a decrease in blood flow to the microcirculation unless fluid intake is increased. Because the hematocrit and hemoglobin are elevated, a transfusion is inappropriate, although transfusions may be needed after the emergent phase once the patient's fluid balance has been restored. On admission to a burn unit, the urine output would be monitored more often than every 4 hours; likely every 1 hour. DIF: Cognitive Level: Apply (application) REF: 257

Which of the following therapeutic measures would be the most effective in treating hypoxemia in the presence of intrapulmonary shunting associated with ARDS? a. Sedating the patient to blunt noxious stimuli b. Increasing the FiO2 on the ventilator c. Administering positiveend expiratory pressure (PEEP) d. Restricting fluids to 500 mL per shift

ANS: C The purpose of using positiveend expiratory pressure (PEEP) in a patient with acute respiratory distress syndrome is to improve oxygenation while reducing FiO to less toxic levels. PEEP has several positive effects on the lungs, including opening collapsed alveoli, stabilizing flooded alveoli, and increasing functional residual capacity. Thus, PEEP decreases intrapulmonary shunting and increases compliance.

The nurse is planning care for a 67-year-old client who recently had abdominal aortic aneurysm repair surgery. The client has developed disseminated intravascular coagulation (DIC). The nurse is aware that the client has an increased risk for what? a. ineffective breathing pattern b. risk for aspiration c. risk for infection d. risk for ineffective cerebral tissue perfusion

ANS: D DIC affects cerebral, cardiopulmonary, and peripheral tissues with clotting that obstructs tissue perfusion. This can result in damage to these tissues. ANALYSIS (Made Easy)

The client who smokes two (2) packs of cigarettes a day develops ARDS after a near drowning. The client asks the nurse, "What is happening to me? Why did I get this?" Which statement by the nurse is most appropriate? a. "Most people who almost drown end up developing ARDS." b. "Platelets and fluid enter the alveoli due to permeability instability." c. "Your lungs are filling up with fluid, causing breathing problems." d. "Smoking has caused your lungs to become weakened, so you got ARDS."

ANS: C This is a basic laypersons terms explanation of ARDS and explains why the client is having trouble breathing. APPLICATION

A nurse is assessing a patient who is receiving a nitroprusside (Nipride) infusion to treat cardiogenic shock. Which finding indicates that the medication is effective? a. No new heart murmurs b. Decreased troponin level c. Warm, pink, and dry skin d. Blood pressure 92/40 mm Hg

ANS: C Warm, pink, and dry skin indicates that perfusion to tissues is improved. Since nitroprusside is a vasodilator, the blood pressure may be low even if the medication is effective. Absence of a heart murmur and a decrease in troponin level are not indicators of improvement in shock. DIF: Cognitive Level: Apply (application) REF: 1644

A patient was admitted to the critical care unit with acute respiratory failure. The patient has been on a ventilator for 3 days and is being considered for weaning. The ventilator highpressure alarm keeps going off. When you enter the room, the ventilator inoperative alarm sounds. Which of the following criteria would indicate that the patient is not tolerating weaning? a. A decrease in heart rate from 92 to 80 beats/min b. An SpO2 of 92% c. An increase in respiratory rate from 22 to 38 breaths/min d. Spontaneous tidal volumes of 300 to 350 mL

ANS: C Weaning intolerance indicators include (1) a decrease in level of consciousness; (2) a systolic blood pressure increased or decreased by 20 mm Hg; (3) a diastolic blood pressure greater than 100 mm Hg; (4) a heart rate increased by 20 beats/min; (5) premature ventricular contractions greater than 6/min, couplets, or runs of ventricular tachycardia; (6) changes in ST segment (usually elevation); (7) a respiratory rate greater than 30 breaths/min or less than 10 breaths/min; (8) a respiratory rate increased by 10 breaths/min; (9) a spontaneous tidal volume less than 250 mL; (10) a PaCO2 increased by 5 to 8 mm Hg or pH less than 7.30; (11) an SpO less than 90%; (12) use of accessory muscles of ventilation; (13) complaints of dyspnea, fatigue, or pain; (14) paradoxical chest wall motion or chest abdominal asynchrony; (15) diaphoresis; and (16) severe agitation or anxiety unrelieved with reassurance.

When SIRS is the result of infection, it is called a. inflammation. b. anaphylaxis. c. sepsis. d. pneumonia.

ANS: C When SIRS is the result of infection, the term sepsis is used.

The gastrointestinal system is a common target organ for MODS related to a. anorexia. b. limited or absent food ingestion. c. disruption of the mucosal barrier from hypoperfusion. d. a decrease in hydrochloric acid secretion.

ANS: C With microcirculatory failure to the gastrointestinal tract, the gut's barrier function may be lost, which leads to bacterial translocation, sustained inflammation, endogenous endotoxemia, and MODS.

Which client would be most at risk for developing disseminated intravascular coagulation (DIC)? a. A 35-year-old pregnant client with placenta previa. b. A 42-year-old client with a pulmonary embolus. c. A 60-year-old client receiving hemodialysis three (3) days a week. d. A 78-year-old cleint diagnosed with septicemia.

ANS: D DIC is a clinical syndrome that develops as a complication of a wide variety of other disorders, with sepsis being the most common cause of DIC. ANALYSIS

The following four patients arrive in the emergency department (ED) after a motor vehicle collision. In which order should the nurse assess them? (Put a comma and a space between each answer choice [A, B, C, D].) a. A 74-year-old with palpitations and chest pain b. A 43-year-old complaining of 7/10 abdominal pain c. A 21-year-old with multiple fractures of the face and jaw d. A 37-year-old with a misaligned left leg with intact pulses

ANS: C, A, B, D The highest priority is to assess the 21-year-old patient for airway obstruction, which is the most life-threatening injury. The 74-year-old patient may have chest pain from cardiac ischemia and should be assessed and have diagnostic testing for this pain. The 43-year-old patient may have abdominal trauma or bleeding and should be seen next to assess circulatory status. The 37-year-old appears to have a possible fracture of the left leg and should be seen soon, but this patient has the least life-threatening injury. DIF: Cognitive Level: Analyze (analysis) REF: 1676

The emergency department (ED) nurse is initiating therapeutic hypothermia in a patient who has been resuscitated after a cardiac arrest. Which actions in the hypothermia protocol can be delegated to an experienced licensed practical/vocational nurse (LPN/LVN) (select all that apply)? a. Continuously monitor heart rhythm. b. Check neurologic status every 2 hours. c. Place cooling blankets above and below patient. d. Give acetaminophen (Tylenol) 650 mg per nasogastric tube. e. Insert rectal temperature probe and attach to cooling blanket control panel.

ANS: C, D, E Experienced LPN/LVNs have the education and scope of practice to implement hypothermia measures (e.g., cooling blanket, temperature probe) and administer medications under the supervision of a registered nurse (RN). Assessment of neurologic status and monitoring the heart rhythm require RN-level education and scope of practice and should be done by the RN. DIF: Cognitive Level: Apply (application) REF: 15-16

Which of the following will cause a nurse to suspect that a patient's pulmonary contusion is worsening? a. A pulmonary artery catheter showing a central venous pressure of 6 cm H2O and a wedge pressure of 8 mm Hg b. An increased need for pain medication c. An arterial blood gas value that demonstrates respiratory alkalosis d. Increased peak airway pressures on the ventilator

ANS: D A contusion manifests initially as a hemorrhage followed by alveolar and interstitial edema. The edema can remain rather localized in the contused area or can spread to other lung areas. Inflammation affects alveolar-capillary units. As more units are affected by inflammation, further pathophysiologic events can occur, including decreased compliance, increased pulmonary vascular resistance, and decreased pulmonary blood flow. These processes result in a ventilation-perfusion imbalance that results in progressive hypoxemia and poor ventilation over a 24- to 48-hour period.

When rewarming a patient who arrived in the emergency department (ED) with a temperature of 87° F (30.6° C), which assessment indicates that the nurse should discontinue active rewarming? a. The patient begins to shiver. b. The BP decreases to 86/42 mm Hg. c. The patient develops atrial fibrillation. d. The core temperature is 94° F (34.4° C).

ANS: D A core temperature of 89.6° F to 93.2° F (32° C to 34° C) indicates that sufficient rewarming has occurred. Dysrhythmias, hypotension, and shivering may occur during rewarming and should be treated but are not an indication to stop rewarming the patient. DIF: Cognitive Level: Apply (application) REF: 1686

The client has been placed on a high level of oxygen. The nurse anticipate that this will cause the clients lungs to do what? a. Improve oxygen uptake b. Increased carbon dioxide levels c. Stabilize carbon dioxide levels d. Reduce the amount of functional alveolar surface area

ANS: D Oxygen toxicity causes direct pulmonary trauma, reducing the amount of alveolar surface area available for gaseous exchange, which results in increased carbon dioxide levels and decreased oxygen uptake. APPLICATION (Made Easy)

A patient with chronic obstructive pulmonary disease (COPD) arrives in the emergency department complaining of shortness of breath and dyspnea on minimal exertion. Which assessment finding by the nurse is most important to report to the health care provider? a. The patient has bibasilar lung crackles. b. The patient is sitting in the tripod position. c. The patient's respirations have decreased from 30 to 10 breaths/minute. d. The patient's pulse oximetry indicates an O2 saturation of 91%.

ANS: D A decrease in respiratory rate in a patient with respiratory distress suggests the onset of fatigue and a high risk for respiratory arrest. Therefore immediate action such as positive pressure ventilation is needed. Patients who are experiencing respiratory distress frequently sit in the tripod position because it decreases the work of breathing. Crackles in the lung bases may be the baseline for a patient with COPD. An oxygen saturation of 91% is common in patients with COPD and will provide adequate gas exchange and tissue oxygenation. DIF: Cognitive Level: Apply (application) REF: 1659

A patient who has burns on the arms, legs, and chest from a house fire has become agitated and restless 8 hours after being admitted to the hospital. Which action should the nurse take first? a. Stay at the bedside and reassure the patient. b. Administer the ordered morphine sulfate IV. c. Assess orientation and level of consciousness. d. Use pulse oximetry to check the oxygen saturation.

ANS: D Agitation in a patient who may have suffered inhalation injury might indicate hypoxia, and this should be assessed by the nurse first. Administration of morphine may be indicated if the nurse determines that the agitation is caused by pain. Assessing level of consciousness and orientation is also appropriate but not as essential as determining whether the patient is hypoxemic. Reassurance is not helpful to reduce agitation in a hypoxemic patient. DIF: Cognitive Level: Apply (application) REF: 458 | 464-465

The correct procedure for selecting an oropharyngeal airway is to: a. measure from the tip of the nose to the ear lobe. b. measure from the mouth to the ear lobe. c. measure from the tip of the nose to the middle of the trachea. d. measure the airway from the corner of the patient's mouth to the angle of the jaw.

ANS: D An oropharyngeal airway's proper size is selected by holding the airway against the side of the patient's face and ensuring that it extends from the corner of the mouth to the angle of the jaw. If the airway is improperly sized, it will occlude the airway. Nasopharyngeal airways are measured by holding the tube against the side of the patient's face and ensuring that it extends from the tip of the nose to the ear lobe.

Which action will the nurse include in the plan of care for a patient in the rehabilitation phase after a burn injury to the right arm and chest? a. Keep the right arm in a position of comfort. b. Avoid the use of sustained-release narcotics. c. Teach about the purpose of tetanus immunization. d. Apply water-based cream to burned areas frequently.

ANS: D Application of water-based emollients will moisturize new skin and decrease flakiness and itching. To avoid contractures, the joints of the right arm should be positioned in an extended position, which is not the position of comfort. Patients may need to continue the use of opioids during rehabilitation. Tetanus immunization would have been given during the emergent phase of the burn injury. DIF: Cognitive Level: Apply (application) REF: 468-469

A patient has been admitted to the critical care unit with the diagnosis of acute respiratory distress syndrome (ARDS). Arterial blood gasses (ABGs) revealed an elevated pH and decreased PaCO2. The patient is becoming fatigued, and the health care provider orders a repeat ABG. The nurse anticipates the following results a. elevated pH and decreased PaCO2 b. elevated pH and elevated PaCO2 c. decreased pH and decreased PaCO2 d. decreased pH and elevated PaCO2

ANS: D Arterial blood gas analysis reveals a low PaO2 despite increases in supplemental oxygen administration (refractory hypoxemia). Initially, the PaCO2 is low as a result of hyperventilation, but eventually the PaCO2 increases as the patient fatigues. The pH is high initially but decreases as respiratory acidosis develops.

Which of the following hemodynamic parameters supports the diagnosis of cardiogenic shock? a. Increased right atrial pressure b. Decreased pulmonary artery wedge pressure c. Increased cardiac output d. Decreased cardiac index

ANS: D Assessment of the hemodynamic parameter of patients in cardiogenic shock reveals a decreased cardiac output and a cardiac index less than 2.2 L/min/m2.

When assessing a patient with chronic obstructive pulmonary disease (COPD), the nurse finds a new onset of agitation and confusion. Which action should the nurse take first? a. Notify the health care provider. b. Check pupils for reaction to light. c. Attempt to calm and reorient the patient. d. Assess oxygenation using pulse oximetry.

ANS: D Because agitation and confusion are frequently the initial indicators of hypoxemia, the nurse's initial action should be to assess oxygen saturation. The other actions are also appropriate, but assessment of oxygenation takes priority over other assessments and notification of the health care provider. DIF: Cognitive Level: Apply (application) REF: 1663

A patient with acute respiratory distress syndrome (ARDS) who is intubated and receiving mechanical ventilation develops a right pneumothorax. Which action will the nurse anticipate taking next? a. Increase the tidal volume and respiratory rate. b. Increase the fraction of inspired oxygen (FIO2). c. Perform endotracheal suctioning more frequently. d. Lower the positive end-expiratory pressure (PEEP).

ANS: D Because barotrauma is associated with high airway pressures, the level of PEEP should be decreased. The other actions will not decrease the risk for pneumothorax. DIF: Cognitive Level: Apply (application) REF: 1668-1669

Which assessment information is most important for the nurse to obtain to evaluate whether treatment of a patient with anaphylactic shock has been effective? a. Heart rate b. Orientation c. Blood pressure d. Oxygen saturation

ANS: D Because the airway edema that is associated with anaphylaxis can affect airway and breathing, the oxygen saturation is the most critical assessment. Improvements in the other assessments will also be expected with effective treatment of anaphylactic shock. DIF: Cognitive Level: Apply (application) REF: 1646

The client has sustained the initial phase of a burn injury. The nurse anticipate that the primary focus of the clients care is: a. Enhancing self-esteem. b. Promoting hygiene. c. Reducing anxiety. d. Preventing infection.

ANS: D Because the bodies protective barrier is damaged and the immune system is compromised, preventing infection is the primary goal. Enhancing self-esteem, promoting hygiene, and reducing anxiety are important but aren't the primary focus. APPLICATION (Made Easy)

The most important aspect of a secondary survey is to a. check circulatory status. b. check electrolyte profile. c. insert a urinary catheter. d. obtain patient history.

ANS: D During the secondary survey, a head-to-toe approach is used to thoroughly examine each body region. The history is one of the most important aspects of the secondary survey. Additional interventions during the resuscitation phase involve placement of urinary and gastric catheters. During resuscitation from traumatic hemorrhagic shock, normalization of standard clinical parameters such as blood pressure, heart rate, and urine output are not adequate. Circulatory status is part of the primary survey.

Which of the following drugs promotes bronchodilation and vasoconstriction? a. SoluMedrol b. Gentamicin c. Atropine d. Epinephrine

ANS: D Epinephrine is given in anaphylactic shock to promote bronchodilation and vasoconstriction and inhibit further release of biochemical mediators.

Which medications are not effective in the immediate treatment of acute anaphylaxis? a. Epinephrine b. Vasopressors c. Diphenhydramine (Benadryl) IV d. Corticosteroids

ANS: D Epinephrine is the firstline treatment of choice for anaphylaxis and should be administered when initial signs and symptoms occur. Several medications are used as secondline adjunctive therapy. Inhaled adrenergic agents are used to treat bronchospasm unresponsive to epinephrine. Diphenhydramine (Benadryl) given 1 to 2 mg/kg (25?0-50 mg) by a slow intravenous line is used to block histamine response. Ranitidine, given in conjunction with diphenhydramine at a dose of 1 mg/kg intravenously over 10 to 15 minutes, has been found helpful. Corticosteroidsare not effective in the immediate treatment of acute anaphylaxis but may be given with the goal of preventing a prolonged or delayed reaction.

A 19-year-old is brought to the emergency department (ED) with multiple lacerations and tissue avulsion of the left hand. When asked about tetanus immunization, the patient denies having any previous vaccinations. The nurse will anticipate giving a. tetanus immunoglobulin (TIG) only. b. TIG and tetanus-diphtheria toxoid (Td). c. tetanus-diphtheria toxoid and pertussis vaccine (Tdap) only. d. TIG and tetanus-diphtheria toxoid and pertussis vaccine (Tdap).

ANS: D For an adult with no previous tetanus immunizations, TIG and Tdap are recommended. The other immunizations are not sufficient for this patient. DIF: Cognitive Level: Apply (application) REF: 1681

Esomeprazole (Nexium) is prescribed for a patient who incurred extensive burn injuries 5 days ago. Which nursing assessment would best evaluate the effectiveness of the medication? a. Bowel sounds b. Stool frequency c. Abdominal distention d. Stools for occult blood

ANS: D H2 blockers and proton pump inhibitors are given to prevent Curling's ulcer in the patient who has suffered burn injuries. Proton pump inhibitors usually do not affect bowel sounds, stool frequency, or appetite. DIF: Cognitive Level: Apply (application) REF: 465

Which nursing interventions included in the care of a mechanically ventilated patient with acute respiratory failure can the registered nurse (RN) delegate to an experienced licensed practical/vocational nurse (LPN/LVN) working in the intensive care unit? a. Assess breath sounds every hour. b. Monitor central venous pressures. c. Place patient in the prone position. d. Insert an indwelling urinary catheter.

ANS: D Insertion of indwelling urinary catheters is included in LPN/LVN education and scope of practice and can be safely delegated to an LPN/LVN who is experienced in caring for critically ill patients. Placing a patient who is on a ventilator in the prone position requires multiple staff, and should be supervised by an RN. Assessment of breath sounds and obtaining central venous pressures require advanced assessment skills and should be done by the RN caring for a critically ill patient. DIF: Cognitive Level: Apply (application) REF: 15-16

The nurse placed a client diagnosed with acute respiratory distress syndrome in the prone position. The nurse determined that this positioning of the client would: a. Improve cardiac output. b. Make the client more comfortable. c. Prevent skin breakdown. d. Recruit more alveoli.

ANS: D Is the pine position may reduce the ability of posterior I'll be all right open and remain open. Turning the client to the prone position may recruit new alveoli in the posterior region of the lawn and improve oxygenation status. Cardiac output shouldn't be affected by the prone position. The prone position doesn't make the client more comfortable, and he often requires the Tatian to tolerate it. Skin breakdown can still look her over the new pressure points. APPLICATION (Made Easy)

The emergency department (ED) triage nurse is assessing four victims involved in a motor vehicle collision. Which patient has the highest priority for treatment? a. A patient with no pedal pulses. b. A patient with an open femur fracture. c. A patient with bleeding facial lacerations. d. A patient with paradoxic chest movements.

ANS: D Most immediate deaths from trauma occur because of problems with ventilation, so the patient with paradoxic chest movements should be treated first. Face and head fractures can obstruct the airway, but the patient with facial injuries only has lacerations. The other two patients also need rapid intervention but do not have airway or breathing problems. DIF: Cognitive Level: Apply (application) REF: 1676

The nurse is reviewing the medication administration record (MAR) on a patient with partial-thickness burns. Which medication is best for the nurse to administer before scheduled wound debridement? a. Ketorolac (Toradol) b. Lorazepam (Ativan) c. Gabapentin (Neurontin) d. Hydromorphone (Dilaudid)

ANS: D Opioid pain medications are the best choice for pain control. The other medications are used as adjuvants to enhance the effects of opioids. DIF: Cognitive Level: Apply (application) REF: 467

The client is receiving positive end-expiratory pressure (PEEP) therapy. The nurse anticipate that the client will exhibit which of the following? a. Bradycardia b. Tachycardia c. Increased blood pressure d. Reduced cardiac output

ANS: D PEEP reduces cardiac output by increasing intrathoracic pressure and reducing the amount of blood delivered to the left side of the heart, they are by reducing cardiac output. It doesn't affect the heart rate, but a decrease in cardiac output may reduce blood pressure, commonly causing a compensatory tachycardia. APPLICATION (Made Easy)

When the nurse educator is evaluating the skills of a new registered nurse (RN) caring for patients experiencing shock, which action by the new RN indicates a need for more education? a. Placing the pulse oximeter on the ear for a patient with septic shock b. Keeping the head of the bed flat for a patient with hypovolemic shock c. Increasing the nitroprusside (Nipride) infusion rate for a patient with a high SVR d. Maintaining the room temperature at 66° to 68° F for a patient with neurogenic shock

ANS: D Patients with neurogenic shock may have poikilothermia. The room temperature should be kept warm to avoid hypothermia. The other actions by the new RN are appropriate. DIF: Cognitive Level: Apply (application) REF: 1634 | 1636

Patients with leftsided pneumonia may benefit from placing them in which of the following positions? a. Reverse Trendelenburg b. Supine c. On the left side d. On the right side

ANS: D Patients with unilateral lung disease should be positioned with the healthy lung in a dependent position. Because gravity normally facilitates preferential ventilation and perfusion to the dependent areas of the lungs, the best gas exchange would take place in the dependent areas of the lungs. Thus, the goal of positioning is to place the least affected area of the patient's lung in the most dependent position. Patients with unilateral lung disease should be positioned with the healthy lung in a dependent position.

Which of the following statements best describes the effects of positivepressure ventilation on cardiac output? a. Positive pressure ventilation increases intrathoracic pressure, which increases venous return and cardiac output. b. Positive pressure ventilation decreases venous return, which increases preload and cardiac output. c. Positive pressure ventilation increases venous return, which decreases preload and cardiac output. d. Positive pressure ventilation increases intrathoracic pressure, which decreases venous return and cardiac output.

ANS: D Positivepressure ventilation increases intrathoracic pressure, which decreases venous return to the right side of the heart. Impaired venous return decreases preload, which results in a decrease in cardiac output.

Preset positive pressure used to augment the patient's inspiratory effort is known as a. positive endexpiratory pressure (PEEP). b. continuous positive airway pressure (CPAP). c. pressure control ventilation (PCV). d. pressure support ventilation (PSV).

ANS: D Preset positive pressure used to augment the patient's inspiratory efforts is known as pressure support ventilation. With continuous positive airway pressure, positive pressure is applied during spontaneous breaths; the patient controls rate, inspiratory flow, and tidal volume. Positive endexpiratory pressure is positive pressure applied at the end of expiration of ventilator breaths.

To evaluate the effectiveness of the pantoprazole (Protonix) ordered for a patient with systemic inflammatory response syndrome (SIRS), which assessment will the nurse perform? a. Auscultate bowel sounds. b. Palpate for abdominal pain. c. Ask the patient about nausea. d. Check stools for occult blood.

ANS: D Proton pump inhibitors are given to decrease the risk for stress ulcers in critically ill patients. The other assessments also will be done, but these will not help in determining the effectiveness of the pantoprazole administration. DIF: Cognitive Level: Apply (application) REF: 1646

Aspiration can best be prevented by a. observing the amount given in the tube feeding. b. assessing the patient's level of consciousness. c. encouraging the patient to cough and to breathe deeply. d. positioning a patient in a semirecumbent position.

ANS: D Semirecumbency has been shown to decrease the risk of aspiration and inhibit the development of hospital associated pneumonia.

Supplemental oxygen administration is usually effective in treating hypoxemia related to a. physiologic shunting. b. dead space ventilation. c. hypercapnia with a PaCO2 of 35 mm Hg. d. ventilation/perfusion mismatching.

ANS: D Supplemental oxygen administration is effective in treating hypoxemia related to alveolar hypoventilation and ventilation/perfusion mismatching. When intrapulmonary shunting exists, supplemental oxygen alone is ineffective. In this situation, positive pressure is necessary to open collapsed alveoli and facilitate their participation in gas exchange. Positive pressure is delivered via invasive and noninvasive mechanical ventilation. If the patient is also experiencing hypercapnia, the PaCO2 will be greater than 45 mm Hg. In patients with chronically elevated PaCO2 levels, these criteria must be broadened to include a pH less than 7.35.

The client with ARDS is on a mechanical ventilator. Which intervention should be included in the nursing care plan addressing the endotracheal Tube (ET) care? a. Do not move or touch the ET tube. b. Obtain a chest x-ray daily. c. Determine if the ET cuff is deflated. d. Ensure that the ET tube is secure.

ANS: D The ET tube should be secured to ensure it does not enter the right main bronchus. The ET tube should be 1 inch above the bifurcation of the bronchi. SYNTHESIS

Which of the following arterial blood gas values would indicate a need for oxygen therapy? a. PaO2 of 80 mm Hg b. PaCO2 of 35 mm Hg c. HCO3 of 24 mEq d. SaO2 of 87%

ANS: D The amount of oxygen administered depends on the pathophysiologic mechanisms affecting the patient's oxygenation status. In most cases, the amount required should provide an arterial partial pressure of oxygen (PaO2) of greater than 60 mm Hg or an arterial hemoglobin saturation (SaO2) of greater than 90% during both rest and exercise.

During the primary survey of a patient with severe leg trauma, the nurse observes that the patient's left pedal pulse is absent and the leg is swollen. Which action will the nurse take next? a. Send blood to the lab for a complete blood count. b. Assess further for a cause of the decreased circulation. c. Finish the airway, breathing, circulation, disability survey. d. Start normal saline fluid infusion with a large-bore IV line.

ANS: D The assessment data indicate that the patient may have arterial trauma and hemorrhage. When a possibly life-threatening injury is found during the primary survey, the nurse should immediately start interventions before proceeding with the survey. Although a complete blood count is indicated, administration of IV fluids should be started first. Completion of the primary survey and further assessment should be completed after the IV fluids are initiated. DIF: Cognitive Level: Apply (application) REF: 1676

A 54-year-old patient arrives in the emergency department (ED) after exposure to powdered lime at work. Which action should the nurse take first? a. Obtain the patient's vital signs. b. Obtain a baseline complete blood count. c. Decontaminate the patient by showering with water. d. Brush off any visible powder on the skin and clothing.

ANS: D The initial action should be to protect staff members and decrease the patient's exposure to the toxin by decontamination. Patients exposed to powdered lime should not be showered; instead any/all visible powder should be brushed off. The other actions can be done after the decontamination is completed. DIF: Cognitive Level: Apply (application) REF: 1690

A patient who was found unconscious in a burning house is brought to the emergency department by ambulance. The nurse notes that the patient's skin color is bright red. Which action should the nurse take first? a. Insert two large-bore IV lines. b. Check the patient's orientation. c. Assess for singed nasal hair and dark oral mucous membranes. d. Place the patient on 100% oxygen using a non-rebreather mask.

ANS: D The patient's history and skin color suggest carbon monoxide poisoning, which should be treated by rapidly starting oxygen at 100%. The other actions can be taken after the action to correct gas exchange. DIF: Cognitive Level: Apply (application) REF: 455

The nurse is caring for a 78-year-old patient who was hospitalized 2 days earlier with community-acquired pneumonia. Which assessment information is most important to communicate to the health care provider? a. Scattered crackles bilaterally in the posterior lung bases. b. Persistent cough that is productive of blood-tinged sputum. c. Temperature of 101.5° F (38.6° C) after 2 days of IV antibiotic therapy. d. Decreased oxygen saturation to 90% with 100% O2 by non-rebreather mask.

ANS: D The patient's low SpO2 despite receiving a high fraction of inspired oxygen (FIO2) indicates the possibility of acute respiratory distress syndrome (ARDS). The patient's blood-tinged sputum and scattered crackles are not unusual in a patient with pneumonia, although they do require continued monitoring. The continued temperature elevation indicates a possible need to change antibiotics, but this is not as urgent a concern as the progression toward hypoxemia despite an increase in O2 flow rate. DIF: Cognitive Level: Apply (application) REF: 1665-1670

Eight hours after a thermal burn covering 50% of a patient's total body surface area (TBSA) the nurse assesses the patient. Which information would be apriority to communicate to the health care provider? a. Blood pressure is 95/48 per arterial line. b. Serous exudate is leaking from the burns. c. Cardiac monitor shows a pulse rate of 108. d. Urine output is 20 mL per hour for the past 2 hours.

ANS: D The urine output should be at least 0.5 to 1.0 mL/kg/hr during the emergent phase, when the patient is at great risk for hypovolemic shock. The nurse should notify the health care provider because a higher IV fluid rate is needed. BP during the emergent phase should be greater than 90 systolic, and the pulse rate should be less than 120. Serous exudate from the burns is expected during the emergent phase. DIF: Cognitive Level: Apply (application) REF: 461

A young adult patient who is in the rehabilitation phase after having deep partial-thickness face and neck burns has a nursing diagnosis of disturbed body image. Which statement by the patient indicates that the problem is resolving? a. "I'm glad the scars are only temporary." b. "I will avoid using a pillow, so my neck will be OK." c. "I bet my boyfriend won't even want to look at me anymore." d. "Do you think dark beige makeup foundation would cover this scar on my cheek?"

ANS: D The willingness to use strategies to enhance appearance is an indication that the disturbed body image is resolving. Expressing feelings about the scars indicates a willingness to discuss appearance, but not resolution of the problem. Because deep partial-thickness burns leave permanent scars, a statement that the scars are temporary indicates denial rather than resolution of the problem. Avoiding using a pillow will help prevent contractures, but it does not address the problem of disturbed body image. DIF: Cognitive Level: Apply (application) REF: 469

After receiving change-of-shift report on a medical unit, which patient should the nurse assess first? a. A patient with cystic fibrosis who has thick, green-colored sputum b. A patient with pneumonia who has crackles bilaterally in the lung bases c. A patient with emphysema who has an oxygen saturation of 90% to 92% d. A patient with septicemia who has intercostal and suprasternal retractions

ANS: D This patient's history of septicemia and labored breathing suggest the onset of ARDS, which will require rapid interventions such as administration of oxygen and use of positive pressure ventilation. The other patients should also be assessed as quickly as possible, but their assessment data are typical of their disease processes and do not suggest deterioration in their status. DIF: Cognitive Level: Analyze (analysis) REF: 1659 | 1667

A 70-year-old client who spilled hot coffee on his lap three days ago has multiple blisters. While the nurse is providing care for the client, two of the blisters break. What is the most appropriate action by the nurse? a. Remove the raised skin because the blister has been compromised. b. Wash the area vigorously with soap and water. c. Apply alcohol to the area. d. Clean the area with normal saline solution and cover it with a dressing.

ANS: D To maintain a sepsis, the nurse should clean the area with normal Celine solution and cover it with the dressing. Removing the raised skin would cost for their skin damage. Washing the area vigorously with soap and water would damage the tissue and called drying. Alcohol would dry out the tissue. APPLICATION (Made Easy)

A patient was admitted to the critical care unit after a left pneumonectomy. The patient is receiving 40% oxygen via a simple facemask. The morning chest radiography study reveals right lower lobe pneumonia. After eating breakfast, the patient suddenly vomits and aspirates. The first action that should be taken after the patient's aspiration event is a. lavaging his airway with normal saline. b. placing him on his back in a semiFowler position. c. administering manual ventilations with a resuscitation bag. d. suctioning his airway.

ANS: D When aspiration is witnessed, emergency treatment should be instituted to secure the airway and minimize pulmonary damage. The patient's head should be turned to the side, and the oral cavity and upper airway should be suctioned immediately to remove the gastric contents.

During the emergent phase of burn care, which assessment will be most useful in determining whether the patient is receiving adequate fluid infusion? a. Check skin turgor. b. Monitor daily weight. c. Assess mucous membranes. d. Measure hourly urine output.

ANS: D When fluid intake is adequate, the urine output will be at least 0.5 to 1 mL/kg/hour. The patient's weight is not useful in this situation because of the effects of third spacing and evaporative fluid loss. Mucous membrane assessment and skin turgor also may be used, but they are not as adequate in determining that fluid infusions are maintaining adequate perfusion. DIF: Cognitive Level: Apply (application) REF: 460-461

The nurse is assessing a 30-year-old client admitted to the emergency department with a deep partial-thickness burn on his arm after a fire in the workplace. The nurse document the assessment findings as: a. Pain and redness. b. Minimal damage to the epidermis. c. Necrotic tissue through all layers of skin. d. Necrotic tissue through most of the dermis.

ANS: D A deep partial-thickness burn causes necrosis of the epidermal and dermal layers. Redness and pain are characteristics of a superficial injury. Superficial burns cause slight epidermal damage. Necrosis through all skin layers is seen with full-thickness injuries. APPLICATION (Made Easy)


Set pelajaran terkait

Business Skills for Technical Professionals, Chapter 7, Minimizing Stress and Avoiding Burnout, Chapter 9, Solving and Preventing Incidents and Problems - Chapter 6, Handling Difficult Customer Situations - Chapter 5

View Set

ATI Scope and Standards of Practice

View Set

Macro - 3.2 Shifts in Demand and Supply for Goods and Services

View Set

Monopolistic Competition/ Oligopoly

View Set